NBSTSA CERTIFICATION EXAM/CERTIFICATION EXAM NBSTSA 1000 QUESTIONS 2022-2023 ORIGINAL VERSION WITH RATIONALES

1.During a laparoscopic appendectomy, the appendix is perforated, and contents begin spilling into the abdomen. Which of the following should the surgical technologist anticipate as the NEXT step?

A.Suction the fluid from the abdomen.
B.Continue removing the appendix as planned.
C.Place a Kelly clamp over the perforation.
D.Close the wound and terminate the procedure.
A

2.A carotid endarterectomy has just been finished and the dressing is on. The surgical technologist begins removing all sharps and non-disposables from the drapes before pushing everything back. According to best practice, which of the following should be the NEXT step?

A.Push the back table, mayo, and basin back and tear it down.
B.Pull the drapes off and put them on the back table in case they are needed again.
C.Push the back table, mayo, and basin back and keep sterile until the patient is transported to PACU.
D.Leave the back table, mayo, and basin close to patient in case they are needed again.
C

3.Which of the following should be prepared for a mastoid dressing?

A.several squared gauze sponges
B.rolled gauze and an abdominal pad
C.several fluffed sponges and rolled gauze
D.gauze sponges and a Velcro strap
C

4.Which of the following is the FIRST step in applying the pneumatic tourniquet?

A.wrapping the limb sequentially with an Esmarch bandage from distal to proximal
B.applying the pneumatic tourniquet to the limb and immediately inflating it
C.prepping the limb with ChloraPrep, including the determined location of the tourniquet
D.padding the limb where the tourniquet is to be wrapped with Webril
D

5.
When performing a dilation and curettage, which of the following instruments is used to grasp the cervix?

A.Sims
B.Auvard
C.Schroeder
D. Bozeman
C

6.
How many lobes do the right and left lungs have, respectively?

A.three, two
B.two, three
C.three, three
D.two, two
A

7.
When a break in aseptic technique occurs, the surgical technologist should FIRST notify the

A.circulating nurse.
B.contaminating team member.
C.OR supervisor.
D.anesthesia provider.
B

8.
Which of the following pathological conditions may be caused by an excessive production of cerebrospinal fluid in the brain?

A.hydrocephalus
B.encephalocele
C.cranial bifida
D.craniosynostosis
A

9.
If unexpected heavy blood loss occurs during an abdominal procedure, a surgical technologist should request additional

A.lap sponges.
B.Raytec sponges.
C.Kitners.
D.cottonoids.
A

10.
Growth in the length of a long bone occurs at the

A.diaphysis.
B.medullary canal.
C.epiphyseal plate.
D.articular cartilage.
C

11.
In which of the following situations can a DNR order be continued in an unconscious patient for a surgical procedure?

A.The patient’s durable POA requests it be continued.
B.The patient’s family requests it be continued.
C.The surgeon requests its be continued.
D.The CRNA requests it be continued.
A

12.
A surgeon asks for warm irrigation after a right hemicolectomy. Which of the following BEST describes the reason for irrigating at this point in the procedure?

A.to clean out the area of any loose tissue
B.to maintain the core body temperature
C.to eliminate any infection-causing microbes
D.to aid visual inspection for leakage
D

13.
The rate per hour in air exchanges in a standard OR suite is

A.5-10.
B.15-25.
C.30-35.
D.40-50.
B

14.
If there is any question related to handling a specific type of specimen, which of the following departments should be consulted?

A.histology
B.laboratory
C.pathology
D.radiology
C

15.
A patient’s dentures are removed in the operating room. Which of the following is the proper procedure for the care of the dentures?

A.Return them to the patient’s unit and leave them at the nurses’ station.
B.Place them in a labeled cup and keep them with the patient’s chart.
C.Wrap them in a paper towel and give them to the anesthesiologist.
D.Ask the circulator to return them to the patient’s room.
B


16.
A patient is scheduled for laser removal of condyloma acuminata. As a safety precaution, the surgical technologist should have which of the following special equipment open and available?

A.

smoke evacuator
B.

suction tip
C.

pulse lavage
D.

endo-catch bag
A

17.
Apart from trauma cases, which of the following incisions would be BEST for sigmoid surgery?

A.

vertical midline
B.

vertical paramedian
C.

oblique subcostal
D.

lower transverse
B

18.
The kidneys help regulate the blood levels of several ions, including

A.

bicarbonate, water, and protein.
B.

protein, sodium, and bicarbonate.
C.

potassium, protein, and sodium.
D.

sodium, potassium, and calcium.
D

19.
Which of the following is the MOST effective mechanical method of controlling bleeding occurring from needle holes in vessel anastomoses?

A.

pledget
B.

suction
C.

ligature
D.

clamp
A

20.
Which of the following tags indicates the least critical of patients in a triage situation?

A.

yellow
B.

black
C.

red
D.

green
D

21.
During a craniotomy, removal of the bone flap is followed by

A.

incision of the dura mater.
B.

opening the galea aponeurotica.
C.

application of the Raney clips.
D.

repairing the dura mater.
A

22.
After gowning and gloving, a surgeon activates the DuraPrep applicator and performs the initial skin prep for surgery. Which of the following is the BEST action to take at this time?

A.

Assist the surgeon in draping the patient.
B.

Regown and reglove the surgeon.
C.

Pass off Bovie and suction.
D.

Inform the supervisor.
B

23.
A specimen obtained for frozen section is generally removed from the sterile field intraoperatively because it

A.

needs to be labeled by the circulator.
B.

is sent to pathology immediately.
C.

will contaminate the sterile field.
D.

needs to be placed in formalin solution.
B

24.
A surgeon is preparing to close the abdomen. Which of the following closing supplies should the surgical technologist have ready for closing the fascial layer?

A.

0 suture and toothed tissue forceps
B.

7-0 Vicryl and toothed tissue forceps
C.

retention sutures and toothed tissue forceps
D.

staples and Adson with teeth
A

25.
During a procedure, a hole is noticed in the surgical technologist’s glove. What should the surgical technologist do NEXT?

A.

Continue the case.
B.

Change the glove using the open-gloving technique.
C.

Change the glove using the closed-gloving technique.
D.

Break scrub, rescrub, gown, and glove.
B

26.
Which of the following instruments should be included in a transurethral resection of the prostate (TURP) setup?

A.

lithotrite
B.

urethrotome
C.

resectoscope
D.

Randall Stone
C

27.
Which of the following terms describes airway assessment by listening with a stethoscope?

A.

observation
B.

perfusion
C.

auscultation
D.

palpation
C

28.
Which of the following techniques promotes safety and protection of the surgical technologist during the intraoperative phase?

A.

securing the electrosurgical pencil to the drapes with a Kelly clamp after every use
B.

recapping hypodermic needle
C.

establishing a neutral zone
D.

using PPE, except in endoscopic procedures
C

29.
Which of the following verifies that autoclave sterilization has occurred?

A.

chemical indicator
B.

Julian date
C.

heat-sensitive tape
D.

biological indicator
D

30.
Which of the following instruments is used in otorhinolaryngology surgery?

A.

Heaney
B.

Baron
C.

Doyen
D.

Poole
B

31.
An organization that regulates the production of biological sterilization test packs in-house is

A.

CDC.
B.

ANSI.
C.

AAMI.
D.

SDS.
C

32.
For an open hemicolectomy, a surgical technologist is preparing a second mayo stand for the placement of dirty surgical instruments. Which of the following instruments is MOST likely to be placed on the second mayo stand?

A.

Penrose drain
B.

Bakes dilator
C.

Allis forceps
D.

Richardson retractor
C

33.
When loading items in a gravity-displaced steam sterilizer, heavy packs should be placed

A.

at the periphery of the load.
B.

on their sides.
C.

on the top rack.
D.

centered on the middle rack.
A

34.
A liver laceration is identified during an emergency exploratory laparotomy. A surgical technologist should prepare a suture attached to which of the following types of needles?

A.

tapered
B.

reverse cutting
C.

blunt
D.

conventional cutting
C

35.
Which of the following instruments is used to thread a vascular graft from one incision to another?

A.

hemostat
B.

tunneler
C.

DeBakey vascular forceps
D.

Randall Stone forceps
B

36.
Which of the following combination of medications are commonly used during a MAC?

A.

nerve conduction blockades with supplemental analgesics, sedatives, or amnesics
B.

induction agents, anticholinergics, and antiemetics
C.

hemostatics, ringer’s solution, analgesics, amnesics or sedatives
D.

tranquilizers, gastric acid management, or lubricants
A

37.
Which of the following items must the surgical technologist have available after a thyroidectomy?

A.

lap sponges, hemostatic clamps, and electrocautery.
B.

endotracheal tube, hemostatic clamps, and scalpel.
C.

knife handle, scalpel blade, and nasogastric tube.
D.

scalpel, hemostatic clamps, and tracheostomy tube.
D

38.
Which of the following is the name of the avascular strip of tissue at the midline which runs from the xiphoid process to the pubis?

A.

linea semilunaris
B.

linea alba
C.

arcuate line
D.

midaxillary line
B

39.
A three-way Foley catheter is inserted following a transurethral resection of the prostate (TURP) in order to

A.

irrigate and aid in hemostasis.
B.

prevent the patient from getting out of bed.
C.

maintain accurate input and output records.
D.

keep the perineal area dry.
A

40.
A patient is in the supine position for a procedure. After the arms are placed on arm boards, padding is placed under the elbows to decrease pressure on which of the following nerves?

A.

brachial
B.

ulnar
C.

axillary
D.

median
B

41.
Which of the following supplies should be pulled by the surgical technologist for a thoracic procedure?

A.

water-sealed drainage system
B.

passive drainage system
C.

Jackson-Pratt drainage system
D.

Andrews-Pynchon suction
A

42.
The FIRST sign of malignant hyperthermia is

A.

muscle twitching.
B.

change in blood pressure.
C.

unexplained tachycardia.
D.

hemorrhage.
C

43.
Prior to the closure of an arteriotomy, which of the following solutions should the surgical technologist have ready for irrigation of the operative site?

A.

lactated Ringer’s
B.

protamine sulfate
C.

sterile water
D.

heparinized saline
D

44.
Herniation of the rectum through a weakened posterior vaginal wall is known as

A.

a rectocele.
B.

a cystocele.
C.

a varicocele.
D.

an enterocele.
A

45.
If a surgeon requests Gelfoam soaked with thrombin, the method to maintain hemostasis is

A.

thermal.
B.

mechanical.
C.

chemical.
D.

electrical.
C

46.
Immediately before a hemorrhoidectomy, which of the following procedures might be performed on the patient in the OR?

A.

cystoscopy
B.

culdoscopy
C.

proctoscopy
D.

laparoscopy
C

47.
During which of the following phases of infection do pathogens actively replicate, but the host shows no symptoms?

A.

convalescence
B.

prodromal phase
C.

acute phase
D.

incubation
D

48.
Which of the following conditions should be treated FIRST?

A.

obstructed airway
B.

closed cerebral injury
C.

shock due to hemorrhage
D.

sucking chest wound
A

49.
During a partial nephrectomy, the surgeon asks for a large chromic suture and Gelfoam. The surgeon would like the Gelfoam to be cut into pledget squares and loaded onto the suture. The MAIN purpose for the use of the Gelfoam and suture is to

A.

cauterize the tissue.
B.

cause a fibrin clot to form.
C.

seal or close tissue.
D.

topically stop bleeding.
D

50.
Which of the following is a tonsillolith?

A.

a stone within the tonsil
B.

inflammation within the tonsil
C.

removal of the tonsil
D.

an abscess of the tonsil
A

51.
Which of the following should be the MINIMUM exposure time in an immediate use sterilizer for unwrapped instruments?

A.

1 minute
B.

3 minutes
C.

5 minutes
D.

7 minutes
B

52.
The anesthesia care provider (ACP) is gathering supplies to initiate a Bier block for the surgical patient. Which of the following items does the surgical technologist give to the ACP to assist in displacing the blood?

A.

pneumatic tourniquet
B.

stockinette
C.

Esmarch bandage
D.

Coban
C

53.
Which of the following hormones regulates urine concentration based on reabsorption?

A.

ADH
B.

PTH
C.

ANP
D.

GFR
A

54.
A surgeon has just completed an incisional biopsy and would like to have an immediate diagnosis. The MOST appropriate NEXT step is to send the specimen to pathology in a

A.

dry specimen container so that a frozen section can be completed.
B.

container containing normal saline so that a frozen section can be completed.
C.

dry specimen container so that a permanent section can be completed.
D.

specimen container containing normal saline so that a permanent section can be completed.
A

55.
In the break room, a precepting surgical technologist is overheard discussing a student’s lack of instrument knowledge. The precepting surgical technologist just committed

A.

tort.
B.

libel.
C.

malpractice.
D.

slander.
D

56.
Which of the following must be done before sterilizing an instrument with a lumen?

A.

Allow the instrument to dry completely.
B.

Ensure the stylet is still within the lumen.
C.

Inject a small amount of distilled water into the lumen.
D.

Wrap the instrument separately in a peel pack pouch and place in the tray.
C

57.
Which of the following should occur after a Veress needle is inserted?

A.

Place a trocar in the umbilical area.
B.

Provide the camera to the surgeon.
C.

Verify the white balance of the camera.
D.

Connect the insufflation tubing.
D

58.
Prions are unique from other infectious agents because they lack which of the following?

A.

cytoplasm
B.

nucleus
C.

DNA
D.

cell membrane
C

59.
In metastatic cancer, which of the following systems allows for tumors to extend beyond the regional area?

A.

skeletal
B.

lymphatic
C.

endocrine
D.

respiratory
B

60.
For which of the following procedures is Gelfoam MOST commonly used?

A.

TVH
B.

cataract extraction
C.

TURP
D.

myringotomy
D

61.
Which of the following is the receiving nerve end of all transmission?

A.

cell body
B.

dendrite
C.

axon
D.
X
trigger zone
B

62.
A surgical “time out” must be performed prior to

A.

leaving the pre-op holding area.
B.

making the initial incision.
C.

placing the grounding pad.
D.

intubating the patient.
B

63.
The opening in the occipital bone which the spinal cord passes through is the foramen

A.

of Monro.
B.

magnum.
C.

rotundum.
D.

of Bochdalek.
B

64.
When preparing cannulated instruments for steam sterilization, the instrument lumen should be

A.

dry for dynamic air removal and wet for gravity displacement.
B.

wet for dynamic air removal and dry for gravity displacement.
C.

dry for both dynamic air removal and for gravity displacement.
D.

wet for both dynamic air removal and for gravity displacement.
A

65.
When steam comes into an autoclave from an outside source, the steam is heated in the

A.

vacuum system.
B.

chamber.
C.

thermostatic trap.
D.

jacket.
D

66.
Which of the following structures form major parts of the brain?

A.

pituitary gland, thalamus, cerebrum, cerebellum
B.

brain stem, cerebellum, diencephalon, cerebrum
C.

cerebrum, cerebellum, pons, medulla oblongata
D.

brain stem, midbrain, hypothalamus, thalamus
B

67.
Which of the following wound classes is assigned to a surgery in which the bowel was accidently perforated during the procedure?

A.I
B.II
C.III
D.IV
C

68.
A Javid shunt is used for which of the following procedures?

A.

carotid endarterectomy
B.

femoral-popliteal bypass
C.

abdominal aneurysmectomy
D.

saphenous vein ligation
A

69.
During a laparoscopic appendectomy, which of the following types of staplers should the surgical technologist have ready on the back table for removing the appendix and ensuring no spillage of bowel content?

A.

intraluminal
B.

linear
C.

skin
D.

purse-string
B

70.
During a transurethral resection of the prostate, bleeding is controlled by

A.

Gelfoam.
B.

irrigation.
C.

cauterization.
D.

suture ligature.
C

71.
During the intraoperative phase, which of the following activities can be completed by both sterile and non-sterile members of the surgical team?

A.

handling of the specimen
B.

maintaining the patient’s operative record
C.

clearing residual blood from the surgical field
D.

maintaining the sterile field and neutral zone
A

72.
When using an ultrasonic cleaner, which of the following should be done FIRST?

A.

Visible debris and biologic matter should be removed.
B.

Instruments should be closed and assembled.
C.

All instruments should be checked to be the same metal type.
D.

Cleaning solution should be changed.
A

73.
Which of the following important considerations should be taken by the surgical technologist when it comes to the suture material for third intention closure?

A.

A low-capillary action suture will not retain moisture.
B.

Choosing the proper tensile strength of the suture can aid in healing.
C.

The handling of suture is more important than the suture material.
D.

The size of the suture is more important than the suture material.
A

74.

Which of the following is the name of the biological indicator used for hydrogen peroxide gas plasma?

A.

Geobacillus stearothermophilus

B.

Bacillus atrophaeus

C.

Staphylococcus aureus

D.

Escherichia coli

B

75.
Which of the following is MOST likely to be used when applying a sterile dressing to a skin graft recipient site?

A.
bolster
B.
ABD pad
C.
4x4s
D.
Steri-Strips
A

76.
Which of the following surgical instruments will a surgeon use in dissection of a breast tumor?

A.

Potts scissors
B.

electrosurgical unit
C.

Metzenbaum scissors
D.

11 blade

C

Which of the following is the opening in the occipital bone through which the spinal cord passes?
A. foramen ovale
B. foramen magnum
C. foramen rotundum
D. foramen of Bochdalek
B. foramen magnum

Sterile supplies and instruments for a case have been opened in OR 1. A surgical technologist was just informed that the case has been moved to OR 2 and the next case in OR 1 is scheduled in two hours. Which of the following is the BEST course of action?

A. Cover the sterile field with a sterile drape and move it from OR 1 to OR 2.
B. New sterile supplies and instruments should be obtained for OR 2 and the setup in OR 1 should be disposed of.
C. OR 1 should be left so that these supplies and instruments can be used by the next case.
D. The surgical technologist should argue that the procedure be held in OR 1 as planned.
B. New sterile supplies and instruments should be obtained for OR 2 and the setup in OR 1 should be disposed of.

Which of the following is the body’s local tissue reaction to injury?
A. fever
B. infection
C. inflammation
D. immune reaction
C. inflammation

Which of the following instruments is used for a common bile duct exploration?
A. Fogarty clamp
B. Randall forceps
C. Pennington clamp
D. Forester ring forceps
B. Randall forceps

Which of the following is the MOST important consideration when cutting suture?

A. avoiding the knot
B. using sharp scissors
C.opening the scissors wide when cutting
D. tagging the end of suture to be cut with a hemostat
A. avoiding the knot

Which of the following structures are ligated and divided during the cholecystectomy?

A.cystic artery and cystic duct
B.hepatic artery and hepatic duct
C. cystic artery and common bile duct
D. hepatic artery and pancreatic duct
A.cystic artery and cystic duct

Which of the following may be used when applying a sterile dressing to a skin graft site?
A. pressure dressing
B. ABD pad
C. 4x4s
D. Steri-Strips
A. pressure dressing

When performing a dilation and curettage, which of the following instruments is used to grasp the cervix?

A. Sims
B. Auvard
C. Schroeder
D. Bozeman
C. Schroeder

On the way into the operating room, a terminally ill patient tells the nurse, “No matter what happens, it will be okay.” Which of the Five Stages of Grief is the patient MOST likely in?
A. denial
B. bargaining
C. depression
D. acceptance
D. acceptance

Which of the following instruments is used for placement of cottonoid strips during a craniotomy?

A. bayonet forceps
B. Russian forceps
C. Kelly clamp
D. angled DeBakey clamp
A. bayonet forceps

When two individuals in sterile attire pass one another in the sterile field, they should

A. pass back-to-back.
B. both face the wall.
C. pass side-to-side.
D. both face the sterile field.
A. pass back-to-back.

Which of the following is used to oxygenate and circulate blood?
A. Doppler ultrasound
B. defibrillator
C. extracorporeal bypass
D. autotransfuser
C. extracorporeal bypass

The PRIMARY consideration in selecting an appropriate anesthetic agent is the
A. physiologic condition of the patient.
B. length of the procedure.
C. type of the procedure.
D. anxiety level of the patient.
A. physiologic condition of the patient.

If two basins are wrapped together for sterilizing, they should be packaged with one basin
A.stacked inside the other and separated by impervious material.
B.on its side and the other upside down.
C.stacked inside the other and wrapped in a towel.
D.stacked inside the other and separated by a towel.
D.stacked inside the other and separated by a towel.

Which of the following items may be used in positioning a patient for a total hip arthroplasty?
A. fracture table
B. Wilson frame
C. Andrews frame
D. bean bag
D. bean bag

A surgical technologist sees a neighbor being admitted. How should the surgical technologist respond?
A.Acknowledge the neighbor.
B.Reassure the neighbor that everything will be fine.
C.Do not acknowledge the neighbor unless greeted first.
D.Ignore the neighbor, even if greeted.
C. Do not acknowledge the neighbor unless greeted first

Instruments that come in contact with the appendiceal stump during an appendectomy should be

A.placed in a separate basin.
B.wiped with a saline sponge.
C.returned to the Mayo stand.
D. wiped with an alcohol sponge.
A.placed in a separate basin.

Which of the following is the MOST serious complication of a strangulated hernia?
A. shock
B. necrosis
C. infection
D. hemorrhage
B. necrosis

Which of the following techniques is NOT acceptable for draping the patient?
A.Hold the drapes high until directly over the patient.
B.Protect the gloved hands by cuffing the drapes.
C.Readjust the drapes as necessary after placement.
D.Place the drapes on a dry area.
C.Readjust the drapes as necessary after placement.

Which of the following structures is separated first when a paramedian incision is performed?
A. peritoneum
B. transverse abdominis
C. external oblique
D. rectus abdominis
D. rectus abdominis

Which of the following is MOST likely to be used to stabilize a cervical fracture?
A. Kirschner wire
B. Hoffman device
C. Steinmann pins
D. Mayfield clamp
D. Mayfield clamp

During a transurethral resection of the prostate, bleeding is controlled by
A. Gelfoam.
B. irrigation.
C. cauterization.
D. suture ligature.
C. cauterization.

To prevent contamination at the end of a surgical procedure, which of the following should be removed LAST by the surgical technologist in the scrub role?
A. patient’s drapes
B. surgical gloves
C. surgical gown
D. Mayo stand drape
B. surgical gloves

In order to remove a portion of the kidney, the surgeon must enter which of the following?
A. synovial membrane
B. Gerota’s fascia
C. Hesselbach’s triangle
D. suprapleural membrane
B. Gerota’s fascia

Which of the following is the MOST common straight catheter used prior to laparoscopic gynecological surgery in the OR?

A. Foley
B. Pezzer
C. Malecot
D. Robinson
D. Robinson

A Stamey needle is used for which of the following types of surgery?
A. hysterectomy
B. prostatectomy
C. bladder suspension
D. anterior colporrhaphy
C. bladder suspension

Immediately before a hemorrhoidectomy, which of the following procedures might be performed on the patient in the operating room?
A. cystoscopy
B. culdoscopy
C. sigmoidoscopy
D. laparoscopy
C. sigmoidoscopy

Which of the following is a fenestrated drape?

A. a drape with an opening that allows exposure of the operative site
B. a split drape that has two tails
C. a drape used to cover the operating table, instrument table, and body regions
D. an incise drape that is self-adhering
A. a drape with an opening that allows exposure of the operative site

An appendectomy incision closed using suture is considered to be which of the following types of wound closure?

A. primary intention
B. secondary intention
C. tertiary intention
D. third intention
A. primary intention

Which of the following patients is MOST likely to acquire a nosocomial infection?
A. a healthy female patient who is 45 years of age

B. a female patient in labor and delivery
C. a healthy male patient who is 45 years of age
D. a male patient who has diabetes
D. a male patient who has diabetes

31
The appendiceal stump, when inverted, is held in place by which of the following types of suture?
A. mattress
B. traction
C. interrupted
D. purse-string
D. purse-string

Aerobic organisms are BEST characterized as
A. pathogenic.
B. nonpathogenic.
C. oxygen-requiring.
D. non-oxygen-requiring.
C. oxygen-requiring.

The PRIMARY purpose of chest tubes is to

A. irrigate the pleural space.
B. eliminate chest leaks.
C. prevent pulmonary emboli.
D. re-establish negative pressure.
D. re-establish negative pressure.

Which of the following is a surgical instrument designed for retracting?
A. Potts
B. Penfield
C. Lowman
D. Gelpi
D. Gelpi

Which of the following surgical procedures may be used to remove a carcinoma from the head of the pancreas?
A. Heller
B. Whipple
C. Bankart
D. Billroth I
B. Whipple

A sterile item used with an electrosurgical unit is the
A. generator.
B. patient return electrode.
C. active electrode.
D. foot pedal.
C. active electrode

What is the desired effect of atropine sulfate when used as a preoperative medication?
A. drowsiness
B. pain relief
C. decreased anxiety
D. drying of secretions
D. drying of secretions

A postoperative total arthroplasty patient presents to the emergency department with severe throbbing pain, fever, and malaise. Which of the following is the likely diagnosis?
A. osteoarthritis
B. osteoporosis
C. osteomyelitis
D. osteomalacia
C. osteomyelitis

Which of the following is used to stabilize a fractured finger?
A. Rush rods
B. Hagie pins
C. Knowles pins
D. Kirschner wires
D. Kirschner wires

Which of the following is the primary purpose for changing into operating room attire?
A. to standardize uniforms

B. to prevent the of spread of microorganisms
C. to identify operating room personnel
D. to avoid damage to personal attire
B. to prevent the of spread of microorganisms

A surgical technologist is assigned to a room where the patient happens to be his neighbor. To protect patient confidentiality, the technologist should indicate that he knows the patient and
A. continue with the procedure.
B. request to be removed from the room.
C. discuss patient personal medical information.

D. inform the surgeon.
B. request to be removed from the room.

Mannitol (Osmitrol) is used in neurosurgical procedures to
A. prevent bleeding.

B. decrease intracranial pressure.
C. anesthetize the operative site.
D. fight possible postoperative infection.
B. decrease intracranial pressure.

Which of the following lies between the lung and the chest wall?
A. mediastinum
B. peritoneum
C. pleura
D. pericardium
C. pleura

Which of the following is applied as a nonadherent dressing?
A. adaptic
B. collodion
C. steri-strip
D. elastoplast
A. adaptic

A Balfour is used in which of the following procedures?
A. tracheostomy
B. sigmoid resection
C. pulmonary decortication
D. stapedectomy
B. sigmoid resection

If the VII cranial nerve is severed, which of the following is the result?
A. loss of hearing
B. loss of vision
C. facial paralysis
D. impairment of eye muscles
C. facial paralysis

When assisting with the closure of a skin incision and operating the skin stapler, the surgical technologist in the scrub role is placing the staples through which two layers?

A.cuticular and subcuticular
B.subicular and subcutaneous
C.subcuticular and subcutaneous
D.subcutaneous and fascia
A.cuticular and subcuticular

During which of the following surgeries is a Meckel’s diverticulum typically discovered?

A. esophagectomy
B. appendectomy
C. prostatectomy
D. varicocelectomy
B. appendectomy

Which of the following layers are transected when a subcostal flank incision is used for a nephrectomy?
A. pre-peritoneal fat
B. linea alba
C. oblique muscles
D. rectus abdominis muscle
C. oblique muscles

A perforator is used for which of the following procedures?
A. cordotomy
B. craniotomy
C. laminectomy
D. spinal fusion
B. craniotomy

In preparation for a cranial aneurysm, which of the following pieces of equipment would MOST likely be used?

A. microscope
B. cryoprobe
C. Jordan Day drill
D. Crutchfield tongs
A. microscope

During strabismus surgery, which of the following is the first tissue layer that must be incised?
A. iris
B. cornea
C. sclera
D. conjunctiva
D. conjunctiva

When can instrument counts be omitted?

A. if an emergency procedure must be performed
B. when the surgical technologist in the circulator role becomes too busy
C. if instruments are needed in another operating room
D. when the patient is receiving outpatient surgery
A. if an emergency procedure must be performed

In which of the following positions is a patient placed for a right nephrectomy?
A. left lateral
B. right lateral
C. left lateral kidney
D. right lateral kidney
C. left lateral kidney

Which of the following examples illustrates UNSAFE technique for disposing of contaminated sharps?

A. Used suture needles and scalpel blades are removed from the needle counter and individually placed in the biohazard sharps container.
B. Used suture needles and scalpel blades remain in the needle counter and the closed needle counter is placed in the biohazard sharps container.
C. The biohazard sharps container is located as close as possible to the area in which the items were used.
D. Disposable surgical blades are removed from knife handles with a needle holder or other instrument.
A. Used suture needles and scalpel blades are removed from the needle counter and individually placed in the biohazard sharps container.

Why is the right kidney typically several centimeters lower than the left kidney?
A. The liver rests superior and anterior to the right kidney.

B. The liver rests superior and posterior to the right kidney.
C. The liver rests inferior and anterior to the right kidney.
D. The liver rests inferior and posterior to the right kidney.
A. The liver rests superior and anterior to the right kidney.

Which of the following should be the MINIMUM exposure time in a flash sterilizer for unwrapped instruments?
A. 1 minute
B. 3 minutes
C. 5 minutes
D. 7 minutes
B. 3 minutes

If a routine surgical procedure was performed without consent, the surgeon committed
A. assault.
B. battery.
C. malpractice.
D. liability.
B. battery.

If an adult patient refuses a blood transfusion, the surgical staff should

A. not administer blood to the patient.
B. administer blood only in an emergency.
C. administer blood only after sedation.
D. not perform any procedure that may require a transfusion.
A. not administer blood to the patient.

During the intraoperative phase, which of the following activities can be completed by both sterile and non-sterile members of the surgical team?

A.caring for the specimen
B.maintaining the patient’s operative record
C.clearing residual blood from the surgical field
D.maintaining the sterile field and neutral zone
A.caring for the specimen

A lumbar meningocele involves removal of
A. cerebrospinal fluid.
B. lumbar disc.
C. meninges.
D. a fluid-filled sac.
D. a fluid-filled sac.

Wrinkle-free padding is applied to an extremity before application of a tourniquet to avoid

A. skin injuries.
B. improper inflation.
C. excessive blood loss.
D. skeletal injuries.
A. skin injuries

A specimen obtained for frozen section is generally removed from the sterile field intraoperatively because it
A. needs to be labeled by the circulator.
B. is sent to pathology immediately.
C. will contaminate the sterile field.
D. needs to be placed in formalin solution.
B. is sent to pathology immediately.

Which of the following procedures is used to visualize the cystic, hepatic, and common ducts?
A. barium enema
B. cholangiography
C. intravenous pyelography
D. upper gastrointestinal series
B. cholangiography

A scleral buckle procedure repairs
A. astigmatism.
B. detached retinas.
C. corneal scarring.
D. eye muscle contractures.
B. detached retinas.

Which of the following retractors should the surgical technologist have available to provide exposure to the patellar tendon while harvesting the graft for an ACL repair?

A. Senn
B. Richardson
C. Hayes
D. Fukuda
A. Senn

The surgical technologist in the scrub role should remain sterile following which of the following surgical procedures?
A.laparoscopic cholecystectomy
B.carotid endarterectomy
C.total hip arthroplasty
D.tympanoplasty
B.carotid endarterectomy

During dissection of the cystic duct in an open cholecystectomy, which of the following instruments is necessary?
A. Ochsner forceps
B. Jennings retractor
C. Mixter clamp
D. O’Sullivan-O’Connor retractor
C. Mixter clamp

A patient entering the operating room for a laparoscopic cholecystectomy looks jaundiced. Which of the following is the BEST explanation for this?
A. pancreatitis
B. biliary obstruction
C. cholecystitis
D. gastroschisis
B. biliary obstruction

The primary purpose of a local anesthetic is to
A. lower skin temperature.
B. reduce fear and anxiety.

C. block peripheral nerve receptors.
D. decrease bleeding.
C. block peripheral nerve receptors.

Which of the following is the correct skin preparation for a female patient undergoing a right mastectomy with possible axillary dissection?
A. shoulder, upper arm and extending down to the elbow, axilla, chest to the table line, and to the left shoulder
B. base of the neck, shoulder, scapula, chest to midline, and circumference of upper arm down to the elbow

C. entire arm, shoulder, axilla, including the hand
D. chin to umbilicus and laterally to the table line
A. shoulder, upper arm and extending down to the elbow, axilla, chest to the table line, and to the left shoulder

When performing a parotidectomy, which of the following nerves is identified and preserved with the use of a nerve stimulator?
A. facial nerve
B. recurrent laryngeal nerve
C. acoustic nerve
D. vagus nerve
A. facial nerve

Which of the following medications is the anesthesia provider responsible for recording the name, amount, and delivery method during a surgical procedure?
A. thrombin
B. topically applied epinephrine
C. polymyxin B sulfate irrigation
D. propofol
D. propofol

As a surgical technologist begins to prep a female patient for Foley catheterization, a bulge in the vaginal wall is noticed. Which of the following would cause this bulge?

A. cystocele
B. endometriosis
C. leiomyoma
D. ectopic pregnancy
A. cystocele

A surgical technologist enters the OR after scrubbing and notices spots of blood on the OR lights. Which of the following actions is the MOST appropriate?
A. Ignore the spots because they are small.
B. Tell the circulator to obtain replacements and wait at the scrub sink.

C. Ask the circulator to clean the spot and then proceed.
D. Break scrub to clean the lights and help open a new sterile field.
D. Break scrub to clean the lights and help open a new sterile field.

Which of the following instruments should be included in a transurethral resection of the prostate (TURP) setup?
A. lithotrite
B. urethrotome
C. resectoscope
D. Randall
C. resectoscope

Which of the following instruments has teeth?

A. Kocher
B. Mixter
C. Babcock
D. Mosquito
A. Kocher

When entering the abdominal cavity, which of the following BEST describes the order of contact with the layers of tissue?
A. skin, fascia, subcutaneous, peritoneum, muscle
B. skin, subcutaneous, muscle, fascia, peritoneum
C. skin, fascia, peritoneum, subcutaneous, muscle

D. skin, subcutaneous, fascia, muscle, peritoneum
D. skin, subcutaneous, fascia, muscle, peritoneum

If a Kelly clamp is left in a patient who underwent a cholecystectomy, which of the following legal concepts apply?
A. misdemeanor
B. res ipsa loquitur
C. extension doctrine
D. intentional tort
B. res ipsa loquitur

A surgical technologist just opened an instrument set that is wet inside after undergoing steam sterilization. Which of the following is the MOST likely reason for the condensate in the package and how can it be corrected?
A. There are not enough instruments in the tray to allow for revaporization of the condensate. The number of instruments in the tray should be increased and the tray should be reprocessed.
B. The instrument set contained absorbant towels, which soaked up the moisture and prevented efficient drying. The tray should be reprocessed without any towels.

C. The absorbent towels were wrapped too tightly around the instrument tray, so they retained moisture. The tray should be reprocessed with loosely wrapped new towels.
D. Condensate has built up on the steam lines. The surgical technologist should clean the steam lines and reprocess the instrument tray.
C. The absorbent towels were wrapped too tightly around the instrument tray, so they retained moisture. The tray should be reprocessed with loosely wrapped new towels

A surgeon’s preference card indicates the surgeon wears a size L gown and double-gloves with a size 7 inner glove and size 6-1/2 outer glove, and the first assistant wears a size XL gown and double-gloves with a size 7-1/2 inner glove and size 7 outer glove. The first assistant arrives prior to the surgeon. Which of the following is the proper order to place the sterile attire on the back table?

A. L gown, 7 glove, 6-1/2 glove, XL gown, 7-1/2 glove, 7 glove
B. XL gown, 7-1/2 glove, 7 glove, L gown, 7 glove, 6-1/2 glove
C. XL gown, 7 glove, 7-1/2 glove, towel, L gown, 6-1/2 glove, 7 glove
D. L gown, 6-1/2 glove, 7 glove, towel, XL gown, 7 glove, 6-1/2 glove
B. XL gown, 7-1/2 glove, 7 glove, L gown, 7 glove, 6-1/2 glove

Following a surgical procedure, which of the following is the BEST sequence of actions for a surgical technologist to take?

A. Remove sterile drapes, remove gown and gloves, and don a pair of unsterile gloves to aid in the care of the postoperative patient.
B. Remove gown and gloves, don a pair of unsterile gloves to remove drapes from the patient, and remove instruments and supplies from the back table.
C. Remove gown and gloves, don a pair of sterile gloves to remove drapes from patient, and remove gloves and don a pain of unsterile gloves to clean the back table.
D. Remove sterile drapes, remove gown and gloves, and don a pair of sterile gloves to clean back table.
A. Remove sterile drapes, remove gown and gloves, and don a pair of unsterile gloves to aid in the care of the postoperative patient.

Which of the following drugs neutralizes the action of heparin?
A. calcium chloride
B. protamine sulfate
C. lidocaine (Xylocaine)
D. neostigmine (Prostigmin)
B. protamine sulfate

Which of the following wounds is MOST likely in the inflammatory phase of wound healing?

A. a Bankart repair, one week post-procedure
B. a laparotomy incision with a cicatrix
C. an open reduction internal fixation (ORIF) of a finger, two weeks post-procedure
D. an abdominal incision, 20 minutes post-closure
D. an abdominal incision, 20 minutes post-closure

To revascularize the heart muscle of a patient with heart disease, a graft may be anastomosed between which of the following two vessels?
A. internal mammary artery to affected coronary artery

B. subclavian artery to affected coronary artery
C. internal mammary artery to aorta
D. subclavian artery to aorta
A. internal mammary artery to affected coronary artery

A laparotomy drape has been placed on a patient, and unprepped skin on the operative site is exposed. Which of the following is the BEST next step?
A. Apply drapes over unprepped skin.
B. Reposition the drapes closer together.
C. Remove drapes and re-prep.
D. Cover with an adhesive drape.
C. Remove drapes and re-prep

While preparing a sterile field, a surgical technologist secures the Bovie holster to the Mayo stand. The surgeon insists on keeping the Bovie by him toward the head of the patient. Which of the following would be the BEST course of action?

A. Move Bovie holster up by head of patient.
B. Do not move the Bovie holster.
C. Tell the circulator to document the surgeon’s response.
D. Remove the Bovie from the field.
A. Move Bovie holster up by head of patient.

Ratcheted instruments should be
A. soaked in a saline solution.
B. prepared for decontamination with curved tips up.

C. opened for washing and sterilizing.
D. sterilized in the sonic washer.
C. opened for washing and sterilizing.

An effective packaging material used for a Balfour to be steam sterilized

A. permits penetration by the sterilant.
B. allows for condensation that produces wet packs.
C. allows for exposure to peracetic acid.
D. is dictated by the surgeon’s preference.
A. permits penetration by the sterilant.

The parathyroid gland regulates
A. insulin.
B. calcium.
C. sodium.
D. aldosterone.
B. calcium.

Scoliosis is MOST commonly found in which of the following curves of the vertebral column?
A. cervical
B. thoracic
C. lumbar
D. sacral
B. thoracic

Which of the following is the MOST effective mechanical method of controlling bleeding occurring from needle holes in vessel anastomoses?

A. pledget
B. suction
C. ligature
D. clamp
A. pledget

Which of the following should be visualized on an x-ray to determine if a long bone is still growing?
A. periosteum
B. diaphysis
C. medullary canal
D. epiphyseal plate
D. epiphyseal plate

Which of the following procedures for obtaining an informed consent form is appropriate?

A. The patient is asked to sign the consent form after the surgeon has explained the procedure.
B. The surgical technologist in the circulator role is ultimately responsible for obtaining the signed consent form.
C. The patient is asked to read the entire consent form after signing it.
D. The consent form is witnessed by one member of the patient’s family.
A. The patient is asked to sign the consent form after the surgeon has explained the procedure.

Which of the following procedures is used for removal of an embolus?
A. endarterectomy
B. balloon catheterization
C. aneurysmectomy
D. fasciotomy
B. balloon catheterization

Which of the following items should be used to transport a patient in traction?

A. bed
B. stretcher
C. wheelchair
D. fracture table
A. bed

At the conclusion of the surgical procedure, the Mayo stand should

A. remain sterile until the patient leaves the room.
B. be pulled away from the sterile field by the circulator after skin closure.
C. be completely emptied immediately after skin closure.
D. be considered unsterile once the dressing is applied.
A. remain sterile until the patient leaves the room.

Which of the following is used during a tympanoplasty?
A. image intensifier
B. hypothermia unit
C. operating microscope
D. argon laser
C. operating microscope

Removing a cataract by ultrasonic vibration and aspiration is called
A. laser surgery.
B. phacoemulsification.
C. extracapsular extraction.
D. intracapsular extraction.
B. phacoemulsification.

Surgical loupes are used to
A. provide hemostasis.
B. infuse the body cavity.
C. magnify the surgical field.
D. stimulate the circulatory system.
C. magnify the surgical field

A patient undergoing a blood transfusion is having a hemolytic reaction. Which of the following actions should the healthcare providers take FIRST?
A. Administer dantrolene.
B. Stop the transfusion.
C. Monitor urine output.
D. Send blood sample to blood bank.
B. Stop the transfusion

A patient’s dentures are removed in the operating room. Which of the following is the proper procedure for the care of the dentures?
A. Return them to the patient’s unit and leave them at the nurses’ station.

B. Place them in a labeled denture cup and keep them with the patient’s chart.
C. Wrap them in a paper towel and give them to the anesthesiologist.
D. Ask the circulator to return them to the patient’s room.
B. Place them in a labeled denture cup and keep them with the patient’s chart.

Which of the following is associated with secondary intention wound healing?

A. delayed suturing
B. tissue granulation
C. wound dehiscence
D. uncomplicated healing
B. tissue granulation

When inflating a 5-cc balloon on a Foley catheter, which of the following would be the MOST appropriate choice?
A. 3-5 mL of sterile water
B. 3-5 mL of normal saline
C. 8-10 mL of sterile water
D. 8-10 mL of normal saline
C. 8-10 mL of sterile water

During a craniotomy, removal of the bone flap is followed by
A.incision of the dura mater.
B.incision of the galea aponeurotica.
C.application of the Raney clips.
D.suturing of the dura mater.
A.incision of the dura mater

During a laparoscopic appendectomy, which of the following types of staplers should the surgical technologist have ready on the back table for removing the appendix and ensuring no spillage of bowel content?
A. intraluminal
B. linear
C. skin
D. purse-string
B. linear

Which of the following solutions is used to identify diseased areas for a cold conization of the cervix?

A. Lugol’s
B. fluorescein
C. methylene blue
D. Isovue
A. Lugol’s

Which of the following is the correct position for dilation and curettage?
A. Sims
B. supine
C. Kraske
D. lithotomy
D. lithotomy

An RN has just asked a surgical technologist to complete a task that is within the surgical technologist’s scope of practice, but the surgical technologist does not feel comfortable completing the task independently. Which of the following is the surgical technologist’s BEST course of action?

A. Ensure that the RN has documented that the task was delegated to the surgical technologist.
B. Request that the RN provides the appropriate level of supervision during the task to ensure safety.
C. Confirm that the initial orientation asked about the surgical technologist’s competency for the task.
D. Obtain verbal instructions for completing the task from the RN before she leaves the room.
B. Request that the RN provides the appropriate level of supervision during the task to ensure safety.

Which of the following may be used for a cerebral aneurysm?
A. dura hook
B. nerve hook
C. brain spoon
D. rake retractor
C. brain spoon

Which of the following instruments should be included in the set-up for a laminectomy?
A. Hohmann
B. Mayfield
C. Kerrison
D. Bennett
C. Kerrison

A surgical technologist is preparing a fresh specimen for frozen section to be passed off of the field. How should the specimen be prepared to be passed to the circulator?
A. wrap it in a moist sponge
B. remove any suture material that may be attached
C. wrap it in a Telfa pad
D. remove from back table using a hemostat
C. wrap it in a Telfa pad

Which of the following suturing techniques features short lateral stitches that are taken beneath the epithelial layer of skin?
A. purse-string
B. traction
C. mattress
D. subcuticular
D. subcuticular

Which of the following surgical needles is MOST appropriately used in a liver resection?
A. cutting
B. taper
C. blunt
D. trocar
C. blunt

Which of the following is NOT an acceptable technique when placing a patient in the lithotomy position?
A. arms placed on arm boards
B. legs placed in stirrups one at a time
C. both legs placed in stirrups simultaneously
D. hips placed over the lower break of the table
B. legs placed in stirrups one at a time

What is the function of silver sulfadiazine (Silvadene)?
A. antimicrobial
B. hemostasis
C. irrigation
D. anticoagulant
A. antimicrobial

If unexpected heavy blood loss occurs during an abdominal procedure, a surgical technologist should request additional

A. lap sponges.
B. RAY-TEC sponges.
C. Kitners.
D. cottonoids.
A. lap sponges

A surgical technologist is explaining the steps of the ESU circuit to a student. The surgical technologist should explain that the electric current is channeled back to the generator through the
A. power source.
B. active electrode (ESU pencil).
C. patient.

D. dispersive (inactive) electrode.
D. dispersive (inactive) electrode.

Sponges that have been added to the sterile field after a procedure has begun should be counted by the
A. circulator and surgeon.
B. surgical technologist in the scrub role and surgical first assistant.
C. surgeon and surgical first assistant.
D. surgical technologist in the scrub role and the circulator.
D. surgical technologist in the scrub role and the circulator.

During a procedure, a surgical technologist touches an unsterile item with a sterile glove. Which of the following should be done?

A. Change the glove at once.
B. Break scrub and rescrub.
C. Wait until closing to change the glove.
D. Ask the surgeon what to do.
A. Change the glove at once

The oxygen-carrying capacity of red blood cells is a function of
A. albumin.
B. hemoglobin.
C. hematocrit.
D. prothrombin.
B. hemoglobin.

Which of the following procedures uses a trocar?
A. cystoscopy
B. laparoscopy
C. proctoscopy
D. bronchoscopy
B. laparoscopy

A major function of the colon is to

A. absorb water.
B. secrete hormones.
C. secrete digestive enzymes.
D. absorb the products of digestion.
A. absorb water

Hepatitis B is caused by a
A. prion.
B. virus.
C. protozoan.
D. bacterium.
B. virus.

At the conclusion of a surgical case, how would the surgical technologist in the scrub role aid in reducing bioburden on instruments as the first step in the sterilization cycle?
A. hand cleaning
B. disinfection
C. sorting
D. sterilization
A. hand cleaning

A potential surgical complication of an inguinal herniorrhaphy is injury to the
A. phrenic nerve.
B. sciatic nerve.
C. spermatic cord.
D. femoral artery.
C. spermatic cord.

What type of incision is typically used for an open cholecystectomy?
A. McBurney’s
B. Pfannenstiel
C. midline
D. subcostal
D. subcostal

Which of the following pathogens would MOST likely cause a post-operative SSI?

A. Staphylococcus aureus
B. Enterococcus spp.
C. Streptococcus spp.
D. Escherichia coli
A. Staphylococcus aureus

Which of the following must be done before sterilizing an instrument with a lumen?
A. allowing the instrument to dry completely
B. ensuring the stylet is still within the lumen

C. injecting a small amount of distilled water into the lumen
D. wrapping separately in a peel pack pouch and placed in the tray
C. injecting a small amount of distilled water into the lumen

Which of the following is the correct order, from the outermost to the innermost layer, of the tissues that compose the wall of the stomach and small intestine?

  1. submucosa
  2. muscularis
  3. serosa
  4. mucosa
    A. 1, 4, 2, 3
    B. 3, 2, 1, 4
    C. 1, 2, 4, 3
    D. 3, 4, 2, 1
    B. 3, 2, 1, 4

Which of the following is NOT a symptom of shock?
A. tachycardia
B. hypertension
C. cold, clammy skin
D. increased respirations
B. hypertension

When transferring an unconscious, post-operative adult patient from the operating room table to a bed, what is the minimum number of people required for safe transfer?
A. 1
B. 2
C. 3
D. 4
D. 4

Which of the following scissors is MOST likely found in a basic hysterectomy set-up?
A. Potts-Smith
B. Stevens
C. Jorgenson
D. Iris
C. Jorgenson

Which of the following is used to remove clots and tissue from the bladder during transurethral resections (TURPs)?
A. lithotrite
B. asepto syringe
C. Ellik evacuator
D. Fogarty catheter
C. Ellik evacuator

A Javid shunt is used for which of the following procedures?

A. carotid endarterectomy
B. femoral-popliteal bypass
C. abdominal aneurysmectomy
D. saphenous vein ligation
A. carotid endarterectomy

How much 0.25% bupivacaine (Marcaine) should be mixed with 1 mL of 1.0% lidocaine (Xylocaine) to make the solution equal strengths of both medications?
A. 2.0 mL
B. 4.0 mL
C. 5.0 mL
D. 10.0 mL
B. 4.0 mL

Hair removed during preparation for cranial surgery should be

A. discarded by the circulator.
B. sent to pathology.
C. used for potential reimplantation.
D. saved because it is personal property.
B. sent to pathology

Which of the following terms best describes a formal medical process when death occurs in surgery and the surgeon and anesthesia care provider must verify that death has occurred?

A. determination of death
B. rigor mortis
C. postmortem procedures
D. livor mortis
A. determination of death

To help prevent toxicity and vascular events in a patient where polymethylmethacrylate (PMMA) is used, which of the following devices should a surgical technologist have available?
A. pulse lavage
B. mask
C. smoke evacuator
D. cement mixer with charcoal filter
A. pulse lavage

A pneumatic tourniquet may be used for extremity surgery to
A. exsanguinate the extremity.
B. allow the surgeon more operating time.
C. reduce blood loss.
D. create an anesthetic effect.
C. reduce blood loss.

During a craniotomy, which of the following types of bone is entered?

A. flat
B. round
C. long
D. short
A. flat

Which of the following is a procedure for removing excess skin from the face and neck?
A. rhinoplasty
B. rhytidectomy
C. cheiloplasty
D. blepharoplasty
B. rhytidectomy

During an open repair of an indirect hernia, which of the following drains should a surgical technologist have available to pass the surgeon once the spermatic cord has been dissected?
A. Malecot
B. T-tube
C. Jackson-Pratt
D. Penrose
D. Penrose

What color is a nitrous oxide tank?

A. blue
B. gray
C. green
D. orange
A. blue

Which of the following is MOST likely to be in place while transferring a patient from the operating room table to the stretcher?

  1. IV tubing
  2. Fogarty catheter
  3. EKG wires
  4. Foley catheter
    A. 1 and 2 only
    B. 1 and 4 only
    C. 2 and 3 only
    D. 3 and 4 only
    B. 1 and 4 only

Which of the following types of sponges is used directly on the structures of the brain?
A. RAY-TEC
B. cherry
C. Weck-cel
D. cottonoid
D. cottonoid

Which of the following procedures requires an incision in the suprasternal notch?

A. mediastinoscopy
B. bronchoscopy
C. parotidectomy
D. pericardectomy
A. mediastinoscopy

Following an abdominal surgery, the surgeon has just closed the peritoneum. Which of the following layers will be sutured next?
A. fascia
B. muscle
C. subcutaneous
D. subcuticular
A. fascia

During the application of arch bars, the surgeon will MOST likely use which of the following between the teeth and around the bar?

A. 0-1 Dexon
B. 5-0 to 6-0 Vicryl
C. 25- or 26-gauge stainless steel wire
D. 35- or 40-gauge stainless steel wire
C. 25- or 26-gauge stainless steel wire

In which of the following positions should a patient be placed for a low anterior colon resection?
A. dorsal recumbent
B. prone
C. lateral
D. lithotomy
D. lithotomy

A #12 knife blade on a #7 handle is MOST commonly used for what type of surgery?

A. gastrectomy
B. hysterectomy
C. adenoidectomy
D. tonsillectomy
D. tonsillectomy

Which of the following anticoagulants should be given during a vascular procedure to prevent normal blood clotting?

A. heparin sodium
B. protamine sulfate
C. atropine
D. epinephrine
A. heparin sodium

What advantage do hand-held retractors have over self-retaining retractors?
A. less exposure
B. operator dependency
C. lesser fatigue factor
D. dynamic adjustability
D. dynamic adjustability

In which of the following congenital anomalies does the urethra open on the underside of the penis?
A. chordee
B. epispadias
C. phimosis
D. hypospadias
D. hypospadias

Once a surgical technologist dons a gown, the cuffs on the gown are considered unsterile because they are
A. covered by the cuff of the sterile glove.
B. double thickness.

C. absorbant and collect moisture.
D. stretchable and pliable.
C. absorbant and collect moisture.

A physician prescribed 50 milligrams of Demerol. The Demerol is supplied in 100 mg/mL. How many mL of Demerol should be given?

A. 0.5 mL
B. 1.0 mL
C. 1.5 mL
D. 2.0 mL
A. 0.5 mL

A surgeon asks a surgical technologist to place a drop of a mydriatic solution in OD. As a result of this medication, the pupil of the

A. right eye will dilate.
B. left eye will dilate.
C. right eye will constrict.
D. left eye will constrict.
A. right eye will dilate.

A Marshall-Marchetti-Krantz (MMK) requires which of the following skin preparations?

  1. abdominal
  2. vaginal
  3. rectal
  4. extremity

A. 1 and 2 only
B. 1 and 3 only
C. 2 and 4 only
D. 3 and 4 only
A. 1 and 2 only

Which of the following conditions is triggered by an injection of succinylcholine (Anectine) and causes an unusually high fever?
A. osteomyelitis
B. sepsis
C. malignant hyperthermia
D. appendicitis
C. malignant hyperthermia

Which of the following statements regarding surgical scrubbing is NOT true?
A. Two accepted methods of surgical scrubbing are the timed method and the counted brush-stroke method.
B. A vigorous 5-minute scrub with a reliable agent may be as effective as a 10-minute scrub done with less mechanical action.
C. Prolonging a scrub beyond the standard scrub length is effective in decreasing microbe counts.
D. When gloves are removed at the end of a surgical procedure, the hands are contaminated and should be immediately washed.
C. Prolonging a scrub beyond the standard scrub length is effective in decreasing microbe counts.

Which of the following spore-forming microorganisms are used as biological monitors for checking steam sterilization effectiveness for a load of instruments in a rigid container?
A. Bacillus stearothermophilus
B. Bacillus atrophaeus
C. Mycobacterium tuberculosis
D. Streptococcus pyogenes
A. Bacillus stearothermophilus

During a laminectomy, disk material is removed with
A. an osteotome.
B. a Kerrison rongeur.
C. a pituitary rongeur.
D. a periosteal elevator.
C. a pituitary rongeur.

Which of the following retractors is used for a C-section?

A. De Lee
B. Hibbs
C. O’Connor-O’Sullivan
D. Auvard
A. De Lee

A surgical technologist is preparing the incision site for the dressing following a TAH. The patient has developed a hematoma at the incision site. The surgeon has left the room. Which of the following actions should the surgical technologist take?
A. Place a pressure dressing.
B. Cover with Dermabond.
C. Steri-Strip the incision.
D. Have the surgeon return.
D. Have the surgeon return.

Which of the following is a type of herniation that occurs with protrusion of the peritoneal sac and its contents (omentum or abdominal viscera)?
A. epigastric
B. umbilical
C. hypogastric
D. femoral
B. umbilical

Heaney clamps are used MOST frequently for which of the following types of surgery?

A. hysterectomy.
B. lobectomy.
C. cystectomy.
D. gastrectomy.
A. hysterectomy

During a C-section, a surgical technologist passes the surgeon a pair of Metzenbaum scissors and Russian forceps. Which of the following anatomical parts is dissected off the uterus and gently retracted prior to uterine incision?
A. ovaries
B. bladder
C. peritoneum
D. broad ligament
B. bladder

Between each surgical procedure, decontamination of walls involves washing
A. areas that have been splashed with blood or debris.
B. all walls with a disinfectant solution.
C. from the floor up to a 5-foot level with a disinfectant solution.
D. from the floor up to a 5-foot level with a viricidal solution.
A. areas that have been splashed with blood or debris.

Which of the following situations would present a problem when preparing instruments for sterilization using a paper-plastic peel pack?
A. The open end of the paper-plastic peel pack has been sealed with tape.
B. The instruments within the paper-plastic peel pack have been held together with tape to avoid shifting.
C. The instruments should be placed in the paper-plastic peel pack with the rings at the end that was sealed by the manufacturer.
D. A felt-tip marker was used to label the plastic on the paper-plastic peel pack prior to sterilization.
B. The instruments within the paper-plastic peel pack have been held together with tape to avoid shifting.

A patient undergoing a procedure while under general anesthesia begins to experience muscle rigidity, followed by tachycardia, metabolic and respiratory acidosis, and cardiac dysrhythmias. Which of the following is the patient experiencing?
A. septic shock
B. malignant hyperthermia
C. cardiopulmonary arrest
D. pulmonary embolism
B. malignant hyperthermia

Anterior and posterior knee stability is influenced by the dynamics of the

A. cruciate ligaments.
B. joint capsule.
C. patellar tendon.
D. collateral ligaments.
A. cruciate ligaments.

When applying a pneumatic tourniquet preoperatively, which of the following items is used to force blood from an extremity?
A. stockinette
B. Coban
C. tourniquet cuff
D. Esmarch bandage
D. Esmarch bandage

After gowning and gloving, a surgeon activates the DuraPrep applicator and proceeds to prep the patient for surgery. Which of the following is the BEST action to take at this time?
A. Assist the surgeon in draping the patient.
B. Regown and reglove the surgeon.
C. Remove drapes and start over.
D. Inform the supervisor.
B. Regown and reglove the surgeon.

After a lung resection is performed, which of the following BEST describes the placement of the collection unit once it is hooked up to the drainage tubing?
A. at the same level as the insertion tube
B. below the level of the insertion tube
C. above the level of the insertion tube
D. at the level of the patient’s feet
B. below the level of the insertion tube

Which of the following methods of hospital sterilization does not corrode metal and passes through woven materials like steam?
A. ionizing radiation
B. Steris
C. glutaraldehyde
D. ethylene oxide
D. ethylene oxide

Tympanosplasty
Surgical operation performed for the reconstruction of the eardrum (tympanic membrane) and/or small bones of the middle ear (ossicles)

After gowning and gloving, a surgeon activates the Duraprep applicator and proceeds to prep the patient for surgery. What is the best action to take at this time?
Regown and reglove the surgeon

What position is used for a pneumonectomy?
Lateral

What is the kraske position?
Anatomical position in which the patient is prone, with hips flexed and elevated, head and feet down.

Which position is correct for a dilation and curettage?
Lithotomy

When a hernia is strangulated, which instrument set is added to the general abdominal set-up?
Gastrointestinal

What is a fenestrated drape?
A drape with an opening that allows exposure of the operative site

Sterile supplies and instruments for a case have been opened in OR one. A surgical technologist was just informed that the case has been moved to OR two and the next case in OR one is scheduled in two hours. What is the best course of action?
New sterile supplies and instruments should be obtained for OR two and the setup in OR one should be disposed of

What is the first step in applying the pneumatic tourniquet necessary in creating a bloodless field?
Padding the area where the tourniquet is to be wrapped with Webril

We have an expert-written solution to this problem!
For a splenectomy procedure, which clamp should be used?
Satinsky

Surgical loupes are used to:
Magnify the surgical field

Which position is the patient placed in for a right nephrectomy?
Left lateral kidney

Once a surgical technologist dons a gown, the cuffs on the gown are considered unsterile because they are:
Absorbant and collect moisture

Instruments that come in contact with the appendiceal stump during an appendectomy should be:
Placed in a separate bin

Which instrument is used to cross-clamp the intestine during a bowel resection?
Allen

The primary purpose of chest tubes is to:
Re-establish the negative pressure

Traction sutures are used on common duct explorations to:
Hold the duct open

The appendiceal stump, when inverted, is held in place by which of the following types of suture?
Purse-string

Which catheter is most commonly used to perform an embolectomy?
Fogarty

A resectoscope is used in what prostatic approach?
Transurethral

Which retractor is preferred for a groin incision?
Weitlaner

What type of dressing is applied as a nonadherent dressing?
Adaptic

During an excisional biopsy of sentinel nodes, the surgical technologist should prepare specimens using which method?
Placed in separate containers and identify

Immediately before a hemorrhoidectomy, which procedure might be performed on the patient in the operating room?
Sigmoidectomy

When a break in antisecptic technique occurs, the surgical technologist in the scrub role should first notify the:
Contaminating team member

Sponges that have been added to the sterile field after a procedure has begun should be counted by the:
Surgical technologist in the scrub role and the circulator

What is a procedure for removing excess skin from the face and neck?
Rhytidectomy

Which rongeur is used to expand the antrostomy in a Caldwell-Luc procedure?
Kerrison

Which procedure is used to visualize the cystic, hepatic, and common ducts?
Cholangiography

Once the spinous process is exposed during a laminectomy, the next instrument needed for the removal of the lamina is a:
Kerrison rongeur

What should the surgical technologist in the scrub role do during a cardiac emergency?
Remain sterile and keep the table sterile

What is used to stabilize a fractured finger?
Kirschner wires

During the application of arch bars, the surgeon will most likely use what between the teeth and around the bar?
25-gauge or 26-gauge stainless steel wire

Which solution is used to identify diseased areas for a cold conization of the cervix?
Lugol’s

When using power saws on bone, what action best helps prevent bone damage?
Irrigation

Which retractor is used in a C-section?
De Lee

A nasogastric tube (NG) is used to:
Drain the stomach

Which procedure requires an incision in the suprasternal nothc?
Mediastinoscopy

The surgical technologist in the scrub role must keep track of blood loss as well as:
Track amount of irrigation used

Following an abdominal surgery, the surgeon has just closed the peritoneum. Which layer will be sutured next?
Fascia

Which medication is the anesthesia provider responsible for recording the name, amount, and delivery method during a surgical procedure?
Propofol

A surgeon is preparing to close the abdomen of an average weight patient. Which closing supplies should the surgical technologist have ready for closing the fascial layer?
2-0 suture and toothed tissue forceps

While preparing a sterile field, a surgical technologist secures the Bovie holster to the Mayo stand. The surgeon insists on keeping the Bovie by him toward the head of the patient. What would be the best course of action?
Move Bovie holster up by the head of the patient

A surgeon is performing a procedure to repair damage from a gunshot wound to the chest. Which type of drain should the surgical technologist have available at the end of the procedure?
Underwater

A liver laceration is identified during an emergency exploratory laparotomy. A surgical technologist will prepare suture attached to which of the following type of needles?
Blunt

A Stamey needle is used for which type of surgery?
Bladder suspension

What advantage do hand-held retractors have over self-retaining retractors?
Dynamic adjustability

If unexpected heavy blood loss occurs during an abdominal procedure, a surgical technologist should request additional:
Lap sponges

What is most commonly used for wound closure in the presence of infection?
Stainless steel

A surgeon asks for warm irrigation after a GI tract reastomosis. What best describes the reason for irrigation at this point in the procedure?
To aid visual inspection for leakage

In comparison to oral Fahrenheit temperatures, rectal Fahrenheit temperatures are
Approximately 1 degree higher

What is the unexpected occurrence that involves physiologic or psychologic injury or death?
Sentinel event

Following a surgical procedure, what is the best sequence of actions for a surgical technologist to take?
Remove sterile drapes, remove gown and gloves, and don a pair of unsterile gloves to aid in the care of the postoperative patient

To prevent contamination at the end of a surgical procedure, what should be removed last by the surgical technologist in the scrub role?
Surgical drapes

An RN has just asked a surgical technologist to complete a task that is within the surgical technologist’s scope of practice, but the surgical technologist does not feel comfortable completing the task independently. What is the surgical technologist’s best course of action?
Request that the RN provides the appropriate level of supervision during the task to ensure safety

What is necessary for ethylene oxide sterilization to occur?
Humidity

The destruction of organisms in an autoclave is accomplished by the:
Coagulation of cell protein

Ratcheted instruments should be:
Opened for washing and sterilizing

An effective packaging material used for a Balfour to be steam sterilized:
Permits penetration by the sterilant

While removing peel pack instruments from the sterilizer, a surgical technologist notices several of the packages have ruptured. What would most likely be the cause of this?
The air was not evacuated from the pouch

Which layer is transected when a subcostal flank incision is used from a nephrectomy?
Oblique muscles

In order to remove a portion of the kidney, the surgeon must enter what?
Gerota’s fascia

What lies between the lung and the chest wall?
pleura

Where is the gastrocnemius muscle located?
Calf

What is the most serious complication of a strangulated hernia?
Necrosis

During a herniaorraphy, the abdominal area that is bound by the rectus abdominis muscle, the inguinal ligament, and the inferior epigastric vessels is the:
Hesselbach triangle

Scoliosis is most commonly found in which curve of the vertebral column?
Thoracic

When performing a Shirodkar procedure, where is the attention being focused?
Cervix

A patient entering the operating room for a laparoscopic cholecystectomy looks jaundiced. What is the best explanation for this?
Biliary obstruction

Which surgical procedure may be used to remove a carcinoma from the head of the pancreas?
Whipple

A scleral buckle procedure repairs:
Detached retinas

Which procedure treats a congenital obstructed lesion found at the distal end of an infant’s stomach?
Pyloromyotomy

What is the surgical correction of a Colles’ fracture?
The distal radius is fractured and the periosteum will be closed with absorbable sutures

What surgery is a Meckel’s diverticulum typically discovered?
Appendectomy

Which structure is behind and below the base of the penis?
Scrotum

When performing a right hemicolectomy, the surgeon will most likely remove portions of what?
Ascending and transverse colon

What destroys bacterial spores?
Sterilant

A patient who is HIV-positive is having an abdominal procedure. Another patient who is HIV-negative is having an orthopedic procedure. During and subsequent to the cases, the surgical technologist should take:
Standard precautions for both patients

Which wound class is assigned to a bowel case in which the resident perforated during the procedure?
III

What is an example of a wound in the inflammatory phase of wound healing?
An abdominal incision, 20 minutes post-closure

What is the desired effect of atropine sulfate when used as a preoperative medication?
Drying of secretions

Which condition is triggered by an injection of succinylcholine (Anectine) and causes an unusually high fever?
Malignant hyperthermia

The primary purpose of a local anesthetic is to:
Block peripheral nerve receptors

How is propofol (Diprivan) administered?
Intravenously

A physician prescribed 50 mg of Demerol. The Demerol is supplied in 100 mg/mL. How many mL of Demerol should be given?
0.5 mL

A patient presented in the OR without family history related to anesthesia complications. Which complication is of most concern related to the lack of family history?
Malignant hyperthermia

The primary consideration in selecting an appropriate anesthetic agent is the:
Physiologic condition of the patient

Which anticoagulant should be given during a vascular procedure to prevent normal blood clotting?
Heparin sodium

A form of contrast media used within the body that is visible when exposed to x-ray is:
Radiopaque

A sterile item used with an electrosurgical unit is the:
Active electrode

What is the correct skin preparation for a female patient undergoing a right mastectomy with possible axillary dissection?
Shoulder, upper arm and extending down to the elbow, axilla, chest to the table line, and to the left shoulder

When donning gloves using the open-gloving technique, the ungloved hand will touch the:
Folded edge of the cuff of the glove

A pneumatic tourniquet may be used for surgery for:
Reduce blood loss

A device used to measure fluid pressure is known as a:
Manometer

An instrument used for measuring the air capacity of lungs is known as:
Spirometer

A monitoring device that measures and numerically displays the concentration of carbon dioxide in exhaled air is called:
Capnometer

What can be used to remove clots and tissue from the bladder during transurethral resections (TURPs)?
Ellik elevator

Which position should a patient be placed for a low anterior colon resection?
Lithotomy

The position in which one lies flat with the chest down and back up is known as the:
Prone position

In which position would a person be if lying on their back?
Supine

If a patient is lying on their right side, which position are they in?
Left lateral recumbent

In which position is the patient in if they are lying supine with legs separated, flexed, and raised in stirrups
Lithotomy

A patient’s dentures are removed in the operating room. What is the proper procedure for the care of the dentures?
Place them in a labeled denture cup and keep them with the patient’s chart

During a procedure, a surgical technologist notices a hole in his glove. What should be done next?
Change the glove using the open-gloving technique

A patient undergoing a blood transfusion is having a hemolytic reaction. What should the healthcare provider do first?
Stop the transfusion

Which instrument is most likely used to stabilize a cervical fracture?
Mayfield clamp

What procedure may require a postauricular incision?
Mastoidectomy

What is the surgical procedure that removes diseased mastoid air cells?
Mastoidectomy

What is the surgical procedure used to remove adenoids for reasons which include: impaired breathing through the nose, chronic infections, or recurrent earaches
Adenoidectomy

What is the surgical procedure of an incision to the eardrum to relieve pressure or drain fluid?
Myringotomy

During an abdominoperineal resection, which stapling device should the surgical technologist have available to form an anal pouch from the ileum?
GIA

A #12 knife blade on a #7 handle is most commonly used for what type of surgery?
Tonsillectomy

Which instrument is used to grasp the base of the hemorrhoid during a hemorrhoidectomy?
Allis

Which retractor may be used during an open cholecystectomy?
Harrington

What may be used when applying a sterile dressing to a skin graft site?
Pressure dressing

The diagnostic procedure that examines the interior of the respiratory structures is:
Bronchoscopy

What is the endoscopic procedure performed to examine the rectouterine pouch and pelvic viscera by the introduction of a culdoscope through the posterior vaginal wall?
Culdoscopy

What is the procedure where a flexed endoscope is inserted through the mouth or, more rarely, through the nares and into the esophagus?
Esophagoscopy

This procedure is done to examine the mediastinum.
Mediastinoscopy

Removing a cataract by ultrasonic vibration and aspiration is called:
Phacoemulsification

Which procedure involves the entire natural lens of the eye, including the capsule that holds it in place, to be removed?
Intracapsular extraction

This procedure is performed by removing the lens of the eye while the elastic capsule that covers the lens is left partially intact to allow implantation of an intraocular lens.
Extracapsular extraction

What procedure repairs a ruptured abdominal aortic aneurysm?
Excision with graft placement

This is the removal of material on the inside of an artery.
Endarterectomy

What surgical procedure is done as a treatment for high blood pressure in the liver?
Portacaval shunt

What is the first step when performing a myringotomy?
Removal of cerumen from the ear canal

Which sponge can be used directly on the structure of the brain?
Cottonoid

Who is responsible for the proper care and handling of an intraoperative specimen?
Surgical technologist in the scrub role and the circulator

During a procedure, a surgical technologist touches an unsterile item with a sterile glove. What should be done?
Change the glove at once

What is used for sponging around the spinal cord?
Cottonoids

During a craniotomy, removal of the bone flap is followed by:
Incision of the dura mater

What stapler should be used for end-to-end intestinal resection?
EEA

What is the surgical operation to remove the back of one or more vertebrae, usually to give access to the spinal cord or to relieve pressure on nerves?
Laminectomy

This procedure is performed to disable selected pain-conducting tracts in the spinal cord, in order to achieve loss of pain and temperature perception.
Cordotomy

What is the most important consideration when cutting suture?
Avoid cutting the knot

During a laminectomy, disk material is removed with:
Pituitary rongeur

Retention sutures are advantageous when the patient is:
Obese

A three-way Foley catheter is inserted following a TURP in order to:
Irrigate and facilitate hemostasis

What is the best placement area for the electrosurgical grounding pad for a patient in the lithotomy position?
Anterior thigh

What is used to oxygenate and circulate blood?
Extracorporeal bypass

This treatment is used for life threatening cardiac dysrythmias and ventricular fibrillation.
Defibrillator

This instrument is used as a noninvasive test that can be used to estimate blood flow through the blood vessels by bouncing high frequency sound waves off circulating red blood cells.
Doppler ultrasound

This mechanical device is used in collecting and re-infusing blood lost from hemorrhage.
Autotransfuser

When performing a urinary bladder catheterization, it is essential to:
Keep the catheter tip sterile

What is the most common straight catheter used prior to laparoscopic gynecological surgery in the OR?
Robinson

During an anterior cervical fusion, a bone graft is taken in order to fuse the vertebral:
Body

What is high blood pressure?
Hypertension

Which needle is most appropriately used in a liver resection?
Blunt

When performing a parotidectomy, which nerve is identified and preserved with the use of a nerve stimulator?
Facial nerve

This is the surgical incision (removal) of the parotid gland, the major and largest of the salivary glands.
Parotidectomy

A surgical technologist is preparing a fresh specimen for frozen section to be passed off the field. How should the specimen be prepared to be passed to the circulator?
Wrap it in a Telfa pad

During a laser laryngoscopy, what supplies should be available?
Wet sponges, towels, and sterile water

When utilizing a laparoscopic set-up, potential hazards associated with the equipment include:
Fiberoptic beam fire

During a laparoscopic appendectomy, which stapler should the surgical technologist have ready on the back table for removing the appendix and ensuring no spillage of bowel content?
Linear

When assisting with the closure of a skin incision and operating the skin stapler, the surgical technologist in the scrub role is placing the staples through which two layers?
Cuticular and subcuticular

When performing an open breast biopsy with needle localization, the specimen is removed en bloc and the wire is:
Removed with the specimen

When a C-section is performed, which position should a pregnant patient be placed in while lying on the operating room table?
Supine with a wedge under the right hip

Between each surgical procedure, decontamination of walls involves washing:
areas that have been splashed with blood or debris

The most common method for decontaminating operating room floors at the end of each case is to:
Damp mop or wet vacuum with a detergent-disinfectant solution

At the conclusion of the surgical procedure, the Mayo stand should:
Remain sterile until the patient leaves the room

Which of the following is most likely to be in place while transferring a patient from the operating room table to the stretcher?

  1. IV tubing
  2. Fogarty catheter
  3. EKG wires
  4. Foley catheter
    1 and 4 only

Care of a surgical specimen includes placing it:
In a bag or container with formalin

If the parents of a dependent minor requiring emergency surgery cannot be located, the operative consent form for surgery may be signed by:
Two consulting physicians

When exposed to ionizing radiation, staff and patients should wear personal protective equipment containing:
Lead

While speaking to the nurse in the pre-operative area, the patient is crying excessively, pouting, and acting like a child. Which coping mechanism is most likely being displayed?
Regression

Which of the following describes events that would require filing an incident report?

  1. Medication error
  2. Bullying
  3. Cardiac or respiratory arrest
  4. Equipment failure prior to surgery
    1, 2, and 3 only

A surgical technologist has been asked to serve as a preceptor to a new employee in the operating room. If the surgeon becomes irritated or anxious because of the new employee’s lack or experience, what actions should be taken?
Quietly coach the new employee in the background

A pneumatic tourniquet may be used for extremity surgery to:
reduce blood loss

What is not acceptable when draping the patient?
Readjusting the drapes as necessary after placement

What position should a patient be placed for a low anterior colon resection?
Lithotomy

What might a brain spoon be used for?
Cerebral aneurysm

A perforator is used for which procedure?
Craniotomy

During an open repair of an indirect hernia, which drain should a surgical technologist have available to pass the surgeon once the spermatic cord has been dissected?
Penrose

What term refers to an abnormal thoracic curve of the spin referred to as “hunchback”?
Kyphosis

To which portion of the colon is the appendix attached?
Cecum

What is the surgical position commonly used for thyroid and gallbladder surgery?
Reverse Trendelenburg

Which muscle type is striated and voluntary?
Skeletal

What is the medical term for a bunion?
Hallux Varus

Which structure has oral, nasal, and laryngeal divisions?
Pharynx

Who is ultimately responsible for obtaining the surgical informed consent?
Surgeon

Which indicator/monitor provides positive assurance of sterility?
Biological

The enzyme used to soften the zonules of the lens before cataract surgery is:
alpha-chymotrypsin

What is the name of the urinary catheter with a small, curved tapered tip used on patients with urethral strictures?
Coude

The Bowie-Dick test is performed:
daily

Which organism is normal resident flora of the intestinal tract?
Escherichia coli

A partially dislocated joint is called:
sublaxation

What tumor is typically benign, encapsulated, and arises from tissue covering the central nervous system structures?
Meningioma

How many hours must an item submerse in glutaraldehyde to sterilize?
10

Which position is most commonly used for neurosurgical procedures?
Supine

What is the term for thread-like appendages that provide bacteria with motion?
Flagella

How many hours must the steam sterilization biological indicator incubate?
24

According to Maslow’s hierarchy of needs, which is the patient experiencing when (s)he trusts the surgical team’s abilities?
Safety

What is an abnormal tract between two epithelium-lined surfaces that is open at both ends?
Fistula

What postoperative complication is associated with total hip arthroplasty?
Pulmonary embolism

A term referring to a waxy secretion in the external ear canal is:
cerumen

What is the action of antagonist drugs?
Counteract the action of another drug

What is the surgical position frequently used for patients undergoing kidney surgery?
Lateral

What surgical position provides optimal visualization of the lower abdomen or pelvis?
Trendelenburg

Which legal principle applies when the patient is given the wrong dose of the local anesthetic?
Res ipsa loquitor

Which instrument is used by the surgeon to intermittently remove prostatic tissue fragments during a TURP?
Ellik evacuator

Permission for treatment given with full knowledge of the risks is a(n):
Informed consent

What is the recommended maximum time limit for a tourniquet to remain inflated on an adult lower extremity?
1.5 hours

What type of procedure would involve the removal of teeth?
Extraction

What is the medical term for a nose bleed?
Epistaxis

Which portion of the stomach surrounds the lower esophageal sphincter?
Cardia

What is the most common cause of intracerebral hemorrhage?
Hypertension

Another name for the tympanic membrane is the:
eardrum

Which suffix means surgical puncture to remove fluid?
-centesis

What is the medical term for removal of the uterus?
Hysterectomy

Which ossicle of the middle ear covers the oval window?
Stapes

What are spiral-shaped bacteria called?
Spirilli

What type of sponge is tightly rolled cotton tape used by surgeons for blunt dissection?
Kitner

The apron-like structure attached to the greater curvature of the stomach is the:
omentum

What is the name of the condition in which a loop of bowel herniates into the Douglas’ cul-de-sac?
Enterocele

Medication used to dilate the pupil is called:
mydriatics

The surgical pack utilized to create the sterile field should be opened on the:
back table

Which procedure would be listed in the OR schedule for a patient undergoing surgical treatment of uterine fibroids?
Myomectomy

What is the surgical procedure that repairs a defect in the wall of the vagina?
Colporrahaphy

This procedure happens when the cervix opens too early during pregnancy, sometimes causing a miscarriage or premature birth.
Cervical cerclage

What is the procedure that removes all organs from a person’s pelvic cavity?
Pelvic exenteration

Which regulation states that blood and body fluids should be considered infectious?
Standard Precautions

What is the procedure for telescoping the intestines in neonates, requiring immediate surgical intervention?
Intussusception

Which surgical team member determines when the patient can be transported from the OR to the PACU?
Anesthesia provider

What is the first part of the small intestine?
Duodenum

The middle part of the small intestine is the:
Jejunum

The third part of the small intestine that attaches to the colon is known as what?
Ileum

The large intestine extends from the:
cecum to anus

What are the parts of the large intestine, in order?
Ascending colon to transverse colon to descending colon to sigmoid colon, finishing at the rectum

A congenital defect in which the fetal blood vessel between the pulmonary artery and aorta does not close is:
ventricular septal defect

A congenital heart condition involving four abnormalities occurring together, including a defective septum between the ventricles and narrowing of the pulmonary artery, and accompanied by cyanosis, is known as:
tetralogy of fallot

The ductus arteriosus is a blood vessel that allows blood to go around the baby’s lungs before birth. What is the condition in which this does not close?
Patent ductus arteriosus

The agent used to flush an artery to prevent clotting is:
heparin

What is the enzyme in blood plasma that causes the clotting of blood by converting fibrinogen to fibrin?
Thrombin

Which drug is used to reverse the anticoagulant effects of heparin?
Protamine sulfate

When a patient’s blood pressure is 135/81, the 135 refers to:
systolic

This refers to the amount of pressure in arteries during contractions of the heart muscle.
Systolic pressure

What is the number that refers to blood pressure when the heart muscle is between beats?
Diastolic pressure

When using an autoclave, what is the minimum temperature and time of exposure?
121 degrees for 15-20 minutes

What is the medical term referring to the middle of the body?
Medial

Cartilage is what type of tissue?
Connective

What is the minimum exposure time for a flash sterilizer for an unwrapped instrument?
3 minutes

  1. Which of the following is considered the MOST important skill for a surgical technologist?

A. management of specimens
B. management of sponges
C. maintaining an orderly setup
D. ensuring adequate lighting
A. Management of specimens

  1. During the postoperative phase, the personnel primarily responsible for transporting soiled instrumentation to decontamination is the

A. housekeeping staff.
B. anesthesia technician.
C. surgical technologist.
D. central processing technician.
C. Surgical technologist

  1. The surgical incision that allows for the BEST visualization for a Billroth I procedure is

A. McBurney’s.
B. Pfannenstiel.
C. midline.
D. subcostal.
C. Midline

  1. CSF is formed in the lateral ventricles of a patient and may not properly flow into the third ventricle through the foramen of

A. Magendie into the fourth ventricle.
B. Monro into the fourth ventricle.
C. Sylvius into the fourth ventricle.
D. Luschka into the fourth ventricle.
C. Sylvius into the fourth ventricle

  1. During an excisional biopsy of sentinel nodes, the surgical technologist should prepare specimens using which of the following methods?

A. Place on a Raytec to be handed off to the circulator.
B. Separate on Telfa and pass to the circulator.
C. Place in separate containers and identify.
D. Place in specimen container held by the circulator using a hemostat.
C. Place in separate containers and identify

  1. A surgical technologist received the following medications onto the sterile field: 0.9% sodium chloride, bacitracin 1 gm, and 0.25% Marcaine with epinephrine. Which of the following is the MOST appropriate way to label these medications?

A. saline + bacitracin and .25 Marcaine + epinephrine
B. NACL + 1 gm baci and 0.25 marc w/epi
C. saline + baci and 0.25% marc with epi
D. 0.9% saline + 1 gm bacitracin and 0.25% Marcaine with epinephrine
D. 0.9% saline + 1 gm bacitracin and 0.25% Marcaine with epinephrine

  1. A thyroidectomy that was done earlier in the day is being brought back for excessive bleeding from the incision. Which of the following items are needed for this case?

A. Monocryl suture and harmonic scalpel
B. laps, silk ties, and hemoclips
C. Vicryl ties, pledgets, and Alexis
D. nylon suture, Raytec, and Geralds
B. laps, silk ties, and hemoclips

  1. If there is any question related to handling a specific type of specimen, which of the following departments should be consulted?

A. histology
B. laboratory
C. pathology
D. radiology
C. Pathology

  1. The required number of air exchanges per hour in an OR is

A. 5 to 10.
B. 15 to 25.
C. 30 to 35.
D. 40 to 50.
B. 15 to 25

  1. A special-made instrument without a backup falls to the floor during a procedure. Which of the following is the BEST reason to hesitate performing immediate-use sterilization?

A. The sterilizer is used for implantable devices, but not instruments.
B. The manufacturer’s standards for sterilizing the instrument are unknown.
C. The staff feels uncomfortable transporting the unwrapped instrument to the OR after sterilization.
D. There are no wrappers or pouches available for sterilizing the instrument.
B. The manufacturer’s standards for sterilizing the instrument are unknown.

  1. When transferring a thoracotomy patient to recovery, the chest drainage system should be

A. disconnected.
B. placed at the patient’s side.
C. positioned above the patient’s head.
D. maintained below the patient’s chest level.
D. Maintained below the patient’s chest level

12.Which of the following procedures requires an incision in the suprasternal notch?

A. mediastinoscopy
B. bronchoscopy
C. parotidectomy
D. pericardectomy
A. Mediastionscopy

  1. When preparing surgical specimens, in addition to the surgical technologist, who else is responsible for accurately labeling the specimen?

A. surgeon and anesthesiologist
B. circulating nurse and first assistant
C. anesthesiologist and first assistant
D. surgeon and circulating nurse
D. Surgeon and circulating nurse

  1. A T-tube drain may be used following

A. a splenectomy.
B. a choledochotomy.
C. an appendectomy.
D. a herniorrhaphy.
B. a choledochotomy

  1. When assisting with the closure of a skin incision and operating the skin stapler, the surgical technologist places the staples through which two layers?

A. cuticular and subcuticular
B. subicular and subcutaneous
C. subcuticular and subcutaneous
D. subcutaneous and fascia
A. cuticular and subcuticular

  1. A type of regional anesthesia in which a tourniquet is applied and local anesthetic is administered intravenously is called a

A. Bier block.
B. peripheral nerve block.
C. MAC.
D. caudal anesthesia.
A. Bier block

  1. During a craniotomy, which of the following types of bone is entered?

A. flat
B. round
C. long
D. short
A. Flat

  1. A physician prescribed 50 milligrams of Demerol. The Demerol is supplied in 100 mg/mL. How many mL of Demerol should be given?

A. 0.5 mL
B. 1.0 mL
C. 1.5 mL
D. 2.0 mL
A. 0.5 mL

  1. Which of the following is MOST likely to be used during a posterior cervical decompression?

A. Kirschner wire
B. Hoffman device
C. Steinmann pins
D. Mayfield clamp
D. Mayfield clamp

  1. Which of the following is the MOST acceptable technique when placing a patient in the lithotomy position?

A. arms placed on arm boards placed at 110 degrees
B. legs placed in stirrups one at a time
C. both legs placed in stirrups simultaneously
D. hips placed above the lower break of the table
C. both legs placed in stirrups simultaneously

  1. Which of the following supplies should the surgical technologist have available for placement of a long arm cast?

A. Xeroform
B. Webril
C. Telfa
D. Kerlix
B. Webril

  1. How much 0.25% bupivacaine should be mixed with 1 mL of 1.0% lidocaine to make the solution equal strengths of both medications?

A. 2.0 mL
B. 4.0 mL
C. 5.0 mL
D. 10.0 mL
B. 4.0 mL

  1. The appendiceal stump, when inverted, is held in place by which of the following types of sutures?

A. Mattress
B. traction
C. interrupted
D. purse-string
D. purse-string

  1. A surgeon asks for warm irrigation after a right hemicolectomy. Which of the following BEST describes the reason for irrigating at this point in the procedure?

A. to clean out the area of any loose tissue
B. to maintain the core body temperature
C. to eliminate any infection-causing microbes
D. to aid visual inspection for leakage
D. to aid visual inspection for leakage

  1. Which of the following devices should the surgical technologist have delivered to the sterile field for a low anterior resection?

A. GIA linear stapler
B. ligating dividing stapler
C. intraluminal stapler
D. TA-60 stapler
C. intraluminal stapler

  1. Which of the following is reshaped during a total hip arthroplasty?

A. fibula
B. acetabulum
C. iliac crest
D. tibia
B. acetabulum

  1. An organization that regulates the production of biological sterilization test packs in-house is

A. CDC.
B. ANSI.
C. AAMI.
D. SDS.
C. AAMI

  1. Rod-shaped bacteria are called

A. bacilli.
B. cocci.
C. vibrio.
D. spirals.
A. bacilli

  1. A type of passive transport when salt crystals dissolve in water is known as

A. phagocytosis.
B. endocytosis.
C. osmosis.
D. diffusion.
D. diffusion

  1. Utilization of lasers in the operating room requires the use of

A. a HEPA filter mask.
B. a lead apron.
C. double gloves.
D. eye protection.
D. eye protection

  1. During application of a Volar splint, the surgical technologist notices that the splint was not completely dried and has caused a pressure point on the patient. Which of the following should be the surgical technologist’s NEXT step?

A. Allow the splint to remain as-is and allow more time to dry.
B. Notify the nurse and let the nurse handle it.
C. Bring it to the surgeon’s attention and assist in putting on a new splint.
D. Apply a new splint and notify the surgeon when finished.
C. Bring it to the surgeon’s attention and assist in putting on a new splint.

  1. Sponges that have been added to the sterile field after a procedure has begun should be counted by the

A. circulator and surgeon.
B. surgical technologist and surgical first assistant.
C. surgeon and surgical first assistant.
D. surgical technologist and the circulator.
D. surgical technologist and the circulator

  1. Which of the following spore-forming microorganisms are used as biological monitors for checking steam sterilization effectiveness for a load of instruments in a rigid container?

A. Bacillus stearothermophilus
B. Bacillus atrophaeus
C. Mycobacterium tuberculosis
D. Streptococcus pyogenes
A. Bacillus stearothermophilus

  1. After surgery, the scrub tech takes the instruments used in surgery to the decontamination area. How soon should these instruments be washed?

A. within 20 minutes
B. within 10 minutes
C. as soon as they arrive to the decontamination area
D. any time the technologist is available
A. within 20 minutes

35.Which of the following types of laser is typically used for stone fragmentation in the ureter?

A. argon
B. krypton
C. carbon dioxide
D. holmium: YAG
D. holmium: YAG

  1. If a patient is undergoing a mastectomy immediately followed by reconstruction, how many surgical setups should be obtained?

A. One; the reconstruction will immediately follow the mastectomy, so the same instruments can be used without additional interventions.
B. One; the reconstruction will immediately follow the mastectomy, so the same instruments can be used after they are cleaned with sterile water.
C. Two; the reconstruction will immediately follow the mastectomy, but a second set of instruments should be used to prevent seeding.
D. Two; the reconstruction will immediately follow the mastectomy, but it will be done in a different OR so two sets are needed.
C. Two; the reconstruction will immediately follow the mastectomy, but a second set of instruments should be used to prevent seeding.

  1. Which of the following instruments is used to thread a vascular graft from one incision to another?

A. hemostat
B. tunneler
C. DeBakey vascular forceps
D. Randall Stone forceps
B. tunneler

  1. A patient has developed a hematoma at the incision site after the surgeon has left the room. The surgical technologist should

A. place a pressure dressing on the site.
B. cover the site with Dermabond.
C. use a Steri-Strip for the incision site.
D. request the surgeon to return.
D. request the surgeon to return

  1. The surgical technologist hands the surgeon the dressing at the end of a procedure. Which of the following is the BEST method to follow in regard to removing the drape?

A. The circulator applies the tape before the removal of the drape.
B. The circulator assists the surgeon with cleaning the patient before removing the drape.
C. A sterile team member holds the dressing in place while the drape is removed.
D. A sterile team member applies the tape after the drape has been removed.
C. A sterile team member holds the dressing in place while the drape is removed.

  1. Following an abdominal surgery, the surgeon has just closed the peritoneum. Which of the following layers will be sutured NEXT?

A. fascia
B. muscle
C. subcutaneous
D. subcuticular
A. Fascia

  1. Which of the following arteries is commonly used for taking a blood pressure reading?

A. radial
B. femoral
C. brachial
D. popliteal
C. brachial

  1. Which of the following is applied as a nonadherent dressing?

A. Adaptic
B. Dermabond
C. Steri-Strip
D. Elastoplast
A. adaptic

  1. When donning gloves using the open-gloving technique, the ungloved hand will touch the

A. outer side of the cuff of the glove.
B. outer side of the sterile glove.
C. cuffed edge nearest the glove palm.
D. folded edge of the cuff of the glove.
D. folded edge of the cuff of the glove

  1. Necrotizing fasciitis confined to the perineum and scrotum is known as

A. cellulitis.
B. Gerota’s fasciitis.
C. boil.
D. Fournier’s gangrene.
D. Fournier’s gangrene

  1. Where is the gastrocnemius muscle located?

A. arm
B. calf
C. back
D. thigh
B. calf

  1. For a splenectomy procedure, which of the following clamps should be used?

A. Allen
B. Doyen
C. Duval
D. Satinsky
D. satinsky

47.A cystectomy with ileal conduit procedure was performed. The surgical technologist anticipates the need for a stoma

A. pouch to collect body fluids.
B. flange to create an artificial orifice.
C. pouch to heal the wound.
D. collar to provide suction for drainage.
A. pouch to collect body fluids

  1. IUSS sterilizers are typically located in an area

A. that is designated as a restricted sub-sterile room.
B. in the semi-restricted corridor with other equipment.
C. outside of the OR department.
D. inside a designated OR.
A. that is designated as a restricted sub-sterile room

  1. Which of the following surgical needles is MOST appropriate for use in a liver resection?

A. cutting
B. taper
C. blunt
D. trocar
C. Blunt

  1. To help in the calculation of the amount of blood loss, the surgical technologist should

A. weigh the blood-soaked sponges.
B. measure the amount of collected fluid.
C. administer the blood replacement.
D. report the amount of irrigation used.
D. report the amount of irrigation used

  1. During dilation and curettage, after the Auvard weighted vaginal speculum is placed, which of the following instruments is used to grasp the cervix?

A. Hegar dilator
B. uterine sound
C. Sim’s curette
D. single tooth tenaculum
D. single tooth tenaculum

  1. Which of the following is the MOST accurate method to check steam autoclave effectiveness?

A. chemical indicator
B. biological indicator
C. external tape
D. temperature recording device
B. biological indicator

  1. Following a surgical procedure, which of the following is the BEST sequence of actions for a surgical technologist to take?

A. Remove sterile drapes, remove gown and gloves, and don a pair of unsterile gloves to aid in the care of the patient.
B. Remove gown and gloves, don a pair of unsterile gloves to remove drapes from the patient, and remove instruments and supplies from the back table.
C. Remove gown and gloves, don a pair of sterile gloves to remove drapes from the patient, then replace those gloves with a pair of unsterile gloves to clean the back table.
D. Remove sterile drapes, remove gown and gloves, and don a pair of sterile gloves to clean the back table.
A. Remove sterile drapes, remove gown and gloves, and don a pair of unsterile gloves to aid in the care of the patient.

54.Which of the following will place a staff member at risk for work-related musculoskeletal disorders?

A. working in the same posture for prolonged periods
B. making small adjustments to a chair or backrest
C. using four team members to transfer a patient laterally
D. frequent position changes while standing at the sterile field
A. working in the same posture for prolonged periods

  1. While setting up for a surgical procedure, a surgical technologist discovers a puncture in the right surgical glove. If no one is available to assist, the BEST solution for the surgical technologist is to

A. request the circulator to reglove the contaminated hand.
B. ask the circulator for a new set of gloves to perform a closed-gloved technique.
C.ask the circulator for a new gown and gloves to regown and reglove.
D. request another set of gloves to place over the punctured glove.
C.ask the circulator for a new gown and gloves to regown and reglove.

  1. During the application of arch bars, the surgeon will MOST likely use which of the following between the teeth and around the bar?

A. 0-1 Dexon
B. 5-0 to 6-0 Vicryl
C. 24- or 26-gauge stainless steel wire
D. 35- or 40-gauge stainless steel wire
C. 24- or 26-gauge stainless steel wire

  1. Which of the following is necessary for ethylene oxide sterilization to occur?

A. indirect contact
B. gravity displacement
C. humidity
D. steam
C. humidity

  1. Which of the following is the correct skin preparation for a female patient undergoing a right mastectomy with possible axillary dissection?

A. shoulder, upper arm and extending down to the elbow, axilla, chest to the table line, and to the left shoulder
B. base of the neck, shoulder, scapula, chest to midline, and circumference of upper arm down to the elbow
C. entire arm, shoulder, axilla, including the hand
D. chin to umbilicus and laterally to the table line
A. shoulder, upper arm and extending down to the elbow, axilla, chest to the table line, and to the left shoulder

  1. Which of the following is done FIRST when performing a myringotomy?

A. inserting a drainage tube
B. incising the tympanic membrane
C. suctioning fluid from the middle ear
D. removing cerumen from the ear canal
D. removing cerumen from the ear canal

  1. During a laminectomy, disk material is removed with a

A. Cobb elevator.
B. Kerrison rongeur.
C. pituitary rongeur.
D. periosteal elevator.
C. pituitary rongeur

  1. When positioning a patient for lithotomy, care should be taken to avoid injury of the peroneal nerve, which can cause injury to the

A. hip.
B. ankle.
C. foot.
D. elbow.
A. hip

  1. Which of the following is the name of the MOST common BI used for ethylene oxide sterilization?

A. Geobacillus thermoglucosidasius
B. Bacillus atrophaeus
C. Bacillus anthracis
D. Geobacillus stearothermophilus
B. Bacillus atrophaeus

  1. Apart from trauma cases, which of the following incisions would be BEST for sigmoid surgery?

A. vertical midline
B. vertical paramedian
C. oblique subcostal
D. lower transverse
B. vertical paramedian

  1. Which of the following suturing techniques features short lateral stitches that are taken beneath the epithelial layer of skin?

A. purse-string
B. traction
C. mattress
D. subcuticular
D. subcuticular

  1. Which of the following reflexes involves plantar flexion of the foot?

A. Achilles
B. patellar
C. Babinski
D. abdominal
A. achilles

  1. In which of the following procedures should the surgical technologist expect to use cottonoids?

A. abdominal hysterectomy
B. abdominal perineal resection
C. lumbar laminectomy
D. open cholecystectomy
C. lumbar laminectomy

  1. Which of the following advantages do hand-held retractors have over self-retaining retractors?

A. less exposure
B. operator dependency
C. fatigue reduction
D. repositioning ability
D. repositioning ability

  1. A surgical technologist is preparing a fresh specimen for frozen section to be passed off of the field. How should the specimen be prepared to be passed to the circulator?

A. Wrap it in a moist sponge.
B. Remove any suture material that may be attached.
C. Wrap it in a Telfa pad.
D. Remove it from the back table using a hemostat.
C. Wrap it in a telfa pad

  1. Which of the following surgical procedures may be used to remove a carcinoma from the head of the pancreas?

A. Heller
B. Whipple
C. Bankart
D. Billroth I
B. Whipple

  1. The only true indicator of item sterility is

A. mechanical.
B. chemical.
C. administrative.
D. biological.
D. biological

  1. A #12 knife blade on a #7 handle is MOST commonly used for which of the following types of surgery?

A. gastrectomy
B. hysterectomy
C. Adenoidectomy
D. tonsillectomy
D. tonsillectomy

  1. Which of the following terms refers to the transfer of heat from one substance to another by the natural movement of molecules, setting other molecules in motion?

A. amplitude
B. conduction
C. convection
D. insulator
B. conduction

  1. Which of the following important considerations should be taken by the surgical technologist when it comes to the suture material for third intention closure?

A. A low-capillary action suture will not retain moisture.
B. Choosing the proper tensile strength of the suture can aid in healing.
C. The handling of suture is more important than the suture material.
D. The size of the suture is more important than the suture material.
A. a low-capillary action suture will not retain moisture

  1. The parathyroid gland regulates

A. insulin.
B. calcium.
C. sodium.
D. aldosterone.
B. calcium

  1. A pad-like sac or cavity that prevents friction within a joint is known as a

A. bursa.
B. fossa.
C. ganglion.
D. ligament.
A. bursa

  1. Which of the following staplers should be used for low anterior intestinal resection?

A. EEA
B. skin
C. LDS
D. purse string
A. EEA

  1. Which of the following is the name of the avascular strip of tissue at the midline which runs from the xiphoid process to the pubis?

A. linea semilunaris
B. linea alba
C. arcuate line
D. midaxillary line
B. linea alba

  1. A wound that has regrowth of damaged blood vessels and synthesizes scar tissue is in which of the following wound healing phases?

A. inflammatory
B. migratory
C. proliferative
D. maturation
B. migratory

  1. When cleaning instruments, the PH of the water that should be used is

A. 5.0.
B. 6.0.
C. 7.0.
D. 8.0.
C. 7.0

  1. Which of the following is a heavy, curved, tapered needle?

A. Mayo
B. Keith
C. Ferguson
D. Arterial
A. mayo

  1. Once a procedure has concluded, the proper order to remove personal protective equipment is

A. gown, gloves, and then mask.
B. gloves, gown, and then hat.
C. gloves, gown, and then mask.
D. gown, gloves, and then hat.
A. gown, gloves, and then mask

  1. Which of the following is the name of the incision made in a simple open nephrectomy?

A. Gibson
B. low transverse
C. subcostal flank
D. McBurney’s
C. subcostal flank

  1. An excised lipoma was filled with non-purulent serosanguineous fluid. Which of the following should be documented as the wound classification for this procedure?

A. clean wound
B. dirty-infected
C. clean-contaminated
D. contaminated
D. contaminated

  1. Which of the following medications is the anesthesia provider responsible for recording the name, amount, and delivery method during a surgical procedure?

A. thrombin
B. topically applied epinephrine
C. polymyxin B sulfate irrigation
D. propofol
D. propofol

  1. Which of the following can prevent thrombophlebitis in a patient who is considered bariatric?

A. a Bair hugger
B. sequential compression device
C. another OR table to accommodate the patient
D. extra positioning aids
B. sequential compression device

  1. Which of the following staplers is used to divide blood vessels?

A. EEA
B. LDS
C. GIA
D. TA-60
B. LDS

  1. Which of the following cranial nerves’ nuclei are associated with the medulla?

A. trigeminal
B. abducens
C. facial
D. vagus
D. vagus

  1. Mannitol (Osmitrol) is used in neurosurgical procedures to

A. prevent bleeding.
B. decrease intracranial pressure.
C. anesthetize the operative site.
D. fight possible postoperative infection.
B. decrease intracranial pressure

  1. Once a surgical technologist dons a gown, the cuffs on the gown are considered unsterile because they are

A. covered by the inside of the sterile glove.
B. twice as thick as the sleeve.
C. absorbant and collect moisture.
D. stretchable and pliable.
C. absorbant and collect moisture

  1. A patient is having a hemolytic reaction. Which of the following actions should the surgical team take FIRST?

A. Administer dantrolene.
B. Stop the transfusion.
C. Monitor urine output.
D. Send a blood sample to blood bank.
B. stop the transfusion

  1. Which of the following is a type of emergency that does not require Federal assistance and is MOST likely to occur in a larger city?

A. mass casualty event
B. pandemic
C. natural disaster
D. terrorism
A. mass casualty event

  1. Which of the following instruments is found in a basic laparotomy setup?

A. Castroviejo needle holder
B. Mayo scissors
C. Dandy clamp
D. Raney clip applier
B. Mayo scissors

  1. Which of the following instruments is used for placement of cottonoids during a craniotomy?

A. bayonet forceps
B. Russian forceps
C. hemostat clamp
D. DeBakey clamp
A. bayonet forceps

  1. Which of the following structures are ligated and divided during a cholecystectomy?

A. cystic artery and cystic duct
B. hepatic artery and hepatic duct
C. cystic artery and common bile duct
D. hepatic artery and pancreatic duct
A. cystic artery and cystic duct

  1. A surgeon requests a patient to be placed into the lithotomy position for the surgical procedure. Which of the following positioning devices must be obtained?

A. leg stirrups
B. chest rolls
C. bean bags
D. axillary rolls
A. leg stirrups

  1. A patient is in the supine position following a lung resection. Which of the following BEST describes the placement of the collection unit once it is connected to the chest tube?

A. above the level of the thoracic cavity
B. below the level of the thoracic cavity
C. at the level of the patient’s head
D. at the level of the patient’s feet
B. below the level of the thoracic cavity

  1. Which of the following anatomical structures is located just under the diaphragm in the left upper quadrant of the abdomen?

A. stomach
B. appendix
C. gallbladder
D. liver
A. stomach

  1. Which of the following is the MOST important consideration when cutting a suture?

A. avoiding the knot
B. using sharp scissors
C. opening the scissors wide when cutting
D. tagging the end of suture to be cut with a hemostat
A. avoiding the knot

  1. The oxygen-carrying capacity of red blood cells is a function of

A. albumin.
B. hemoglobin.
C. hematocrit.
D. prothrombin.
B. hemoglobin

100.At the start of a surgical procedure, two Forester sponge forceps are clamped to the end of a drape sheet so the drape ends can be securely weighted down during C-arm use. At the end of the surgical case, the two Forester sponge forceps are unclamped by the radiology technician. The surgical technologist should

A. take the sponge forceps and place them on the back table.
B. request that the sponge forceps be placed inside the case cart.
C. rake the sponge forceps and place them in the case cart.
D. request that the sponge forceps be placed onto the back table.
B. request that the sponge forceps be placed inside the case cart

  1. During a procedure, a surgical technologist touches an unsterile item with a sterile glove. The surgical technologist should

A. change the glove at once.
B. break scrub and rescrub.
C. wait until closing to change the glove.
D. ask the surgeon for a recommendation.
A. change the glove at once

  1. During an abdominoperineal resection, which of the following stapling devices should the surgical technologist have available to form an anal pouch from the ileum?

A. TA
B. GIA
C. EEA
D. LDS
B. GIA

  1. In decontamination, the instruments are sticking and difficult to open. To aid in the process of washing instruments, the surgical technologist should have

A. used a basin with sterile water on the field to soak and clean instruments intraoperatively.
B. cleaned the instruments between uses with normal saline.
C. used a basin with enzymatic cleaner on the field to soak and clean instruments intraoperatively.
D. requested the circulator clean the instruments with glycine.
A. used a basin with sterile water on the field to soak and clean instruments intraoperatively

  1. While removing peel pack instruments from the sterilizer, a surgical technologist notices several of the packages have ruptured. The MOST likely cause is that the

A. air was not evacuated from the pouch.
B. temperature of the autoclave was too high.
C. package was sterilized too long.
D. instruments in the pouch were too heavy.
A. air was not evacuated from the pouch

  1. A surgical technologist is explaining the steps of the electrosurgical unit circuit to a student. The surgical technologist should explain that the electric current is channeled back to the generator through the

A. power source.
B. active electrode.
C. patient.
D. dispersive electrode.
D. dispersive electrode

  1. When wrapping individual instruments, what is the MOST important sterilization principle?

A. having an indicator with the instrument
B. using an absorbent towel with the instrument
C. placing a plastic wrap over the instrument
D. using any size wrapper with the instrument
A. having an indicator with the instrument

  1. Preparing instrument sets and packages for sterilization requires a label indicating the

A. sterilization date, sterilizer ID, load number, and preparer’s initials.
B. preparation date, preparer’s initials, and department to which the package belongs.
C. preparer’s initials, manufacturer ID, and cycle number.
D. department code, date of expiration, and sterilizer time parameters.
A. sterilization date, sterilizer ID, load number, and preparer’s initials.

  1. Which of the following portions of the colon is used as an “end colostomy” following an abdominal perineal resection?

A. cecum
B. hepatic flexure
C. sigmoid
D. splenic flexure
C. sigmoid

  1. A resectoscope is used in which of the following prostatic approaches?

A. perineal
B. suprapubic
C. retropubic
D. transurethral
D. transurethral

  1. In which of the following situations can a DNR order be continued in an unconscious patient for a surgical procedure?

A. The patient’s durable POA requests it be continued.
B. The patient’s family requests it be continued.
C. The surgeon requests its be continued.
D. The CRNA requests it be continued.
A. The patient’s durable POA requests it be continued

  1. In local anesthesia, which of the following drugs provides the shortest duration of numbness?

A. lidocaine
B. ropivacaine
C. bupivacaine
D. mepivacaine
A. lidocaine

  1. After completion of a procedure, the dispersive electrode is removed and the skin is examined for

A. burns.
B. cleanliness.
C. excess adhesive.
D. bruises.
A. burns

  1. Retention sutures are typically advantageous when a patient is

A. pediatric.
B. athletic.
C. thin.
D. diabetic.
D. diabetic

  1. Which of the following surrounds the kidney?

A. synovial membrane
B. Gerota’s fascia
C. Hesselbach’s triangle
D. suprapleural membrane
B. Gerota’s fascia

  1. Heaney clamps are used MOST frequently for which of the following types of surgery?

A. hysterectomy.
B. lobectomy.
C. cystectomy.
D. gastrectomy.
A. hysterectomy

  1. During a peripheral vascular procedure, heparinized saline is used as an irrigant. The circulating nurse is ready to draw 10,000 units of heparin from a 2-milliliter ampoule to deliver onto the sterile field, but only 5,000 units of heparin is needed. How much heparin, in milliliters, should be delivered to the sterile field?

A. 0.5 mL
B. 1 mL
C. 1.5 mL
D. 2 mL
B. 1 mL

  1. After the surgical gown has been removed and disposed of in the waste container, how should the surgical gloves be removed?

A. closed glove technique
B. one layer at a time
C. two layers at a time
D. open glove technique
B. one layer at a time

  1. A major function of the colon is to

A. absorb water.
B. secrete hormones.
C. secrete digestive enzymes.
D. absorb the products of digestion.
A. absorb water

  1. Which of the following is the MOST efficient way to communicate changes to a surgeon’s preference card to the surgical technologist staff?

A. phone call
B. email
C. written mail
D. word of mouth
B. email

  1. If an x-ray cassette must be placed directly against the operative site, it must be

A. sterilized using hydrogen peroxide plasma.
B. placed in a sterile pillowcase.
C. sterilized using peracetic acid.
D. placed in a sterile cassette cover.
D. placed in a sterile cassette cover

  1. When steam comes into an autoclave from an outside source, the steam is heated in the

A. vacuum system.
B. chamber.
C. thermostatic trap.
D. jacket.
D. jacket

  1. Which of the following instruments has teeth?

A. Kocher
B. Mixter
C. Babcock
D. Mosquito
A. Kocher

  1. Following a hysterectomy, the surgical technologist participates in the debriefing, which includes

A. stating the patient’s pain level.
B. discharge planning.
C. completeness and correction of all counts.
D. stating the length of time the surgery lasted.
C. completeness and correction of all counts

  1. A surgeon performing an arterial graft placement on a patient confirms that there is an occlusion in a portion of the artery, and that it must be removed. The surgeon requests a 3-way stopcock and a 1 milliliter tuberculin syringe. Which of the following items will the surgical technologist also need to be delivered onto the sterile field for removal of the occlusion?

A. umbilical tape
B. Fogarty catheter
C. Groshong catheter
D. vessel loops
B. Fogarty catheter

  1. A surgical “time out” must be performed prior to

A. leaving the pre-op holding area.
B. making the initial incision.
C. placing the grounding pad.
D. intubating the patient.
B. making the initial incision

  1. Which of the following classifications of disinfection is used to kill all bacteria, viruses, and fungi?

A. standard
B. intermediate-level
C. high-level
D. antiseptic
C. high-level

  1. Which of the following is true regarding Avitene?

A. It must be reconstituted with saline before use.
B. It is a form of mechanical hemostatic used on bone.
C. It is a gelatin sponge commonly soaked in thrombin.
D. It is a collagen powder that must be kept dry.
D. It is a collagen powder that must be kept dry

  1. During general anesthesia, which of the following complications of vomiting is considered the MOST dangerous?

A. dehiscence
B. aspiration
C. dehydration
D. evisceration
B. aspiration

  1. Which of the following types of hernias require urgent surgical treatment?

A. reducible
B. strangulated
C. incisional
D. inguinal
B. strangulated

  1. Which of the following is the receiving nerve end of all transmission?

A. cell body
B. dendrite
C. axon
D. trigger zone
B. dendrite

  1. Which of the following instruments must be readily available for a modified radical mastectomy?

A. Stevens scissors
B. Jorgenson scissors
C. extra #11 blades
D. extra #10 blades
D. extra #10 blades

  1. During knee meniscectomy, the surgeon notices bright spots in the picture. Which of the following is the MOST likely problem?

A. dirty lens
B. camera has condensation
C. loose wire
D. faulty fiber optic cable
D. faulty fiber optic cable

  1. After manually removing bioburden from a Westcott scissor, the process to continue decontamination is called

A. emulsification.
B. cavitation.
C. immersion.
D. deionization.
B. cavitation

  1. Which of the following terms best describes the formal medical process when death occurs in surgery and the surgeon and anesthesia care provider must verify that death has occurred?

A. determination of death
B. rigor mortis
C. postmortem procedures
D. livor mortis
A. determination of death

  1. While opening a back table, the surgical technologist notices a hair in the open sterile pack. The chemical indicators show that the pack has been properly sterilized. The surgical technologist should

A. Ignore it since the hair has gone through the sterilization process.
B. Continue setup and cover with a blue towel.
C. Break down and obtain new equipment.
D. Notify materials management.
C. Break down and obtain new equipment

  1. When inflating a 5-cc balloon on a Foley catheter, which of the following would be the MOST appropriate choice?

A. 3-5 mL of sterile water
B. 3-5 mL of normal saline
C. 8-10 mL of sterile water
D. 8-10 mL of normal saline
C. 8-10 mL of sterile water

  1. The surgical technologist is alarmed by the slurred speech, smell of alcohol, and disheveled appearance of the surgeon while he dons gown and gloves. The surgical technologist should

A. observe the surgeon for further impairment.
B. share the event with co-workers in the lounge.
C. request a different assignment to avoid liability.
D. request a nurse manager to come to the room.
D. request a nurse manager to come to the room

  1. An Esmarch should be applied to an extremity

A. proximal to distal.
B. distal to proximal.
C. ventral to dorsal.
D. dorsal to ventral.
B. distal to proximal

  1. While preparing a sterile field, a surgical technologist secures the Bovie holster to the Mayo stand. The surgeon insists on keeping the Bovie attached to the drapes within reach for ease of access. Which of the following would be the BEST course of action?

A. Move Bovie holster up by head of patient.
B. Do not move the Bovie holster.
C. Tell the circulator to document the surgeon’s response.
D. Remove the Bovie from the field.
A. Move Bovie holster up by head of patient

  1. Which of the following machines in the sterile processing department uses cavitation for cleaning instruments?

A. washer-decontaminator
B. steam sterilizer
C. ethylene oxide sterilizer
D. ultrasonic cleaner
D. ultrasonic cleaner

  1. Which of the following instrument sets should be added to the general abdominal set-up when an inguinal hernia is thought to be strangulated?

A. rectal
B. vascular
C. common duct
D. gastrointestinal
D. gastrointestinal

  1. During an intraocular procedure, there is an increase in a patient’s IOP. Which medication should the surgical technologist anticipate being used?

A. Mydriacyl
B. Pilocarpine
C. Cyclogel
D. Wydase
B. Pilocarpine

  1. Which of the following is an example of a sesamoid bone?

A. patella
B. tibia
C. femur
D. sternum
A. patella

  1. Which of the following is a normal adult white blood cell count?

A. 1,000 to 4,000
B. 5,000 to 10,000
C. 11,000 to 14,000
D. 15,000 to 20,000
B. 5,000 to 10,000

  1. Which of the following is the fourth stage of the Kubler-Ross Model?

A. denial
B. depression
C. acceptance
D. anger
B. depression

  1. Which of the following connections do ligaments make?

A. muscle to muscle
B. muscle to bone
C. tendon to muscle
D. bone to bone
D. bone to bone

  1. When using the neutral zone technique, a surgical technologist should

A. pass the blade to the surgeon hand to hand.
B. place soiled sponges in the neutral zone.
C. ensure communication is established.
D. place multiple sharps in the neutral zone together.
C. ensure communication is established

  1. Which of the following categories of symbiosis applies when both organisms benefit from, and depend upon, one another to a certain extent?

A. commensalism
B. antagonism
C. mutualism
D. parasitism
C. mutualism

  1. Once the spinous process is exposed during a laminectomy, the NEXT instrument needed for the removal of the lamina is a

A. Kerrison rongeur.
B. pituitary rongeur.
C. Freer elevator.
D. periosteal elevator.
A. Kerrison rongeur

  1. Which of the following is true regarding surgical drapes that have been placed on the patient?

A. They can be rearranged during the procedure if they begin to fall off the patient.
B. If incorrectly placed, they can be lifted up and replaced properly.
C. They may not be moved once placed on the patient.
D. After placing a drape on the patient, it should be kept free from blood throughout the procedure.
C. They may not be moved once placed on the patient

1.A complication of steep reverse Trendelenburg position is

A.

deep vein thrombosis.

B.

arterial compression.

C.

cardiac overload.

D.

respiratory alkalosis.

A.

deep vein thrombosis.

Which of the following positions should be utilized for posterior colpotomy?

A.

prone

B.

supine

C.

lithotomy

D.

Fowler’s

C.

lithotomy

Which of the following BEST prevents hypothermia in the OR?

A.

forced air warming blanket

B.

warmed IV solution

C.

warmed irrigation solution

D.

multiple layers of warm blankets

A.

forced air warming blanket

OR humidity must be maintained between which of the following ranges?

A.

5% to 20%

B.

20% to 60%

C.

40% to 80%

D.

50% to 90%

B.

20% to 60%

An important process in which all team members must participate just prior to incision is called the

A.

huddle.

B.

timeout.

C.

debriefing.

D.

consent.

B.

timeout.

Which of the following is Mohs surgery used to treat?

A.

malignant melanoma and basal cell carcinoma

B.

basal cell and squamous cell carcinoma

C.

malignant melanoma and squamous cell carcinoma

D.

basal cell and Merkel cell carcinoma

B.

basal cell and squamous cell carcinoma

When reviewing the chart for a patient scheduled for a mammoplasty, it is noticed that the patient’s K+ is 5.4. The patient is suffering from which of the following?

A.

hyperkalemia

B.

hypokalemia

C.

hypercalcemia

D.

hypocalcemia

A.

hyperkalemia

Which of the following are the preferred positioning devices used during surgical procedures?

A.

foam pads

B.

saline bags

C.

gel pads

D.

sheet rolls

C.

gel pads

When inserting a Foley catheter into a male patient, the surgical first assistant encounters difficulty. Which of the following is the NEXT step that the surgical first assistant should take?

A.

Proceed with force until the catheter goes in.

B.

Stop attempting and notify the surgeon.

C.

Abort the surgical procedure.

D.

Obtain a new kit and try again.

B.

Stop attempting and notify the surgeon.

Which of the following could occur as a result of a tourniquet causing excessive pressure on the nerves?

A.

paralysis

B.

ischemia

C.

hematoma

D.

necrosis

A.

paralysis

Which of the following positions is used for an LAVH?

A.

reverse Trendelenburg

B.

supine

C.

lateral recumbent

D.

lithotomy

D.

lithotomy

Which of the following types of positioning injury occurs when underlying tissue is stationary while the skeletal structure moves?

A.

friction injury

B.

pressure injury

C.

shear injury

D.

negativity injury

C.

shear injury

Which of the following patient positions would be MOST likely to cause an obturator nerve injury?

A.

lateral

B.

prone

C.

lithotomy

D.

semi-Fowler’s

C.

lithotomy

When the surgeon incises tissue, the surgical first assistant usually provides a clear field by

A.

light adjustment.

B.

clamping.

C.

retraction.

D.

cauterizing.

C.

retraction.

Which of the following is known as a Kelly procedure?

A.

anterior colporrhaphy

B.

cervical conization

C.

Bartholin cyst marsupialization

D.

vulvectomy

A.

anterior colporrhaphy

Which of the following is a type of shunt used during a carotid endarterectomy?

A.

Javid

B.

Fogarty

C.

Swan-Ganz

D.

Rummel

A.

Javid

Which of the following organs has a fragile collagen-rich capsule and must be handled carefully?

A.

kidney

B.

gallbladder

C.

liver

D.

spleen

D.

spleen

Which of the following procedures would require a pneumatic tourniquet?

A.

femoral-popliteal bypass

B.

arteriovenous (AV) fistula creation

C.

total knee replacement

D.

intramedullary tibial nailing

C.

total knee replacement

When applying clamps to a major vessel that is to be preserved, which of the following is the preferred clamp to use?

A.

Peon

B.

Crile

C.

Satinsky

D.

Ochsner

C.

Satinsky

Which of the following is a role of the surgical first assistant during robotics cases?

A.

Place access cannula.

B.

Exchange instrument arms.

C.

Position the patient cart.

D.

Operate instruments from console.

B.

Exchange instrument arms.

Which of the following can be done to prevent wrong site surgeries?

A.

Bring the patient to the OR prior to the surgeon marking the site.

B.

Surgical first assistant should mark the patient if the surgeon is not available.

C.

Surgeon must mark the surgical site preoperatively.

D.

Put an “X” or write the word “NO” on the non-operative side.

C.

Surgeon must mark the surgical site preoperatively.

When placing a drain for an abdominoplasty, which of the following is MOST commonly used?

A.

Jackson-Pratt

B.

Malecot

C.

Pleur-Vac

D.

T-tube

A.

Jackson-Pratt

Injury to which of the following arteries is commonly seen in the temporal lobes, where the skull is the thinnest and meningeal blood vessels are numerous, and is the usual cause of epidural hematomas?

A.

middle meningeal

B.

posterior meningeal

C.

temporal

D.

cerebral

A.

middle meningeal

Which of the following is the desired effect when applying a partially threaded cancellous screw to a bone fragment?

A.

distraction

B.

rotation

C.

compression

D.

subluxation

C.

compression

During an abdominal hysterectomy, the surgeon initially isolates the uterus by separating it from the

A.

bladder.

B.

round ligaments.

C.

broad ligaments.

D.

cervix.

B.

round ligaments.

During a carotid endarterectomy, the final occlusion clamp to be removed after anastomosis is on the

A.

common carotid.

B.

internal carotid.

C.

external carotid.

D.

thyroid branch.

B.

internal carotid.

The primary purpose of a robotic device designed to hold and maneuver a laparoscope is to

A.

eliminate tremor associated with operating an endoscope.

B.

replace the surgeon.

C.

decrease set-up time for an endoscope.

D.

reduce the incidence of fogging.

A.

eliminate tremor associated with operating an endoscope.

During a carotid endarterectomy, the arteries are unclamped and reclamped before arteriotomy closure is performed to

A.

flush free debris from the carotid.

B.

restore complete blood flow to the brain.

C.

increase collateral flow.

D.

ensure patency of the carotid body.

A.

flush free debris from the carotid.

When opening the aneurysmal sac during resection of an abdominal aortic aneurysm (AAA), which of the following arteries is ligated?

A.

lumbar

B.

supine mesenteric

C.

renal

D.

common hepatic

A.

lumbar

When entering the skull during a craniotomy, it is important to prevent excess heat to the skull and to avoid floating bone dust by

A.

using a moist sponge to dab at the drill site.

B.

keeping suction close to the drill site.

C.

irrigating at the drill site with saline.

D.

using Raney clips to prevent the drill from getting hot.

C.

irrigating at the drill site with saline.

During arterial embolectomy, which of the following catheters is used for clot removal?

A.

Swan-Ganz

B.

Groshong

C.

Fogarty

D.

Broviac

C.

Fogarty

A 3-way Foley catheter with a 30-cc balloon should be inserted following a transurethral prostate resection (TURP) to

A.

irrigate and facilitate hemostasis.

B.

prevent the patient from getting out of bed.

C.

keep accurate input and output records.

D.

minimize postoperative hypertrophy.

A.

irrigate and facilitate hemostasis.

When should the patient’s chest tubes be attached to the drainage system during a thoracotomy?

A.

after the thoracic cavity is closed

B.

immediately upon insertion

C.

within an hour of the procedure

D.

upon arrival in the PACU

A.

after the thoracic cavity is closed

A 60-year-old obese patient with a history of diabetes and renal insufficiency has undergone drainage of an abdominal abscess through a midline incision. Which of the following should be considered for wound closure?

A.

Multifilament, absorbable suture material provides for optimal wound healing.

B.

A continuous suture line should be used.

C.

An interrupted retention suture should be used.

D.

Use of blunt surgical needles is a consideration in the presence of infection.

C.

An interrupted retention suture should be used.

A 54-year-old female presents to the OR with a 4 cm oblique volar forearm laceration and a 9 cm transverse dorsal forearm laceration from a dog bite. Which of the following suturing methods and materials is the BEST choice for closure of the wound?

A.

interrupted silk

B.

continuous polyglactin

C.

interrupted polypropylene

D.

continuous polyester

C.

interrupted polypropylene

In a previously infected wound that has been opened and debrided, which of the following is the preferred suture material in the event that subcutaneous tissue is closed?

A.

braided absorbable

B.

braided non-absorbable

C.

monofilament absorbable

D.

monofilament non-absorbable

C.

monofilament absorbable

During arterial cannulation in cardiopulmonary bypass (CPB), the cannula tip must be directed into which of the following arteries?

A.

pulmonary

B.

brachiocephalic

C.√

aorta

D.

coronaries

C.

aorta

The primary purpose of a chest tube is to

A.

irrigate the chest.

B.

eliminate leaks in the lungs.

C.

prevent pulmonary emboli.

D.

re-establish negative pressure.

D.

re-establish negative pressure.

During surgical treatment for a cerebral aneurysm, the aneurysm clip is placed at the

A.

neck of the aneurysm.

B.

apex of the aneurysm.

C.

arterial bifurcation.

D.

parent vessel.

A.

neck of the aneurysm.

Which mechanical method of hemostasis should a surgical first assistant use in neurosurgery?

A.

Avitene

B.

thrombin spray

C.

Surgicel

D.

bone wax

D.

bone wax

Which of the following procedures is usually performed in conjunction with a Nissen or Toupet Fundoplication?

A.

sleeve gastrectomy

B.

gastric bypass (Roux-N-Y)

C.

Transoral Incisionless Fundoplication (TIF)

D.

hiatal hernia repair

D.

hiatal hernia repair

A patient with both anterior and posterior knee joint instability and buckling during downhill ambulation is BEST treated by which of the following surgical interventions?

A.

ACL reconstruction

B.

total joint arthroplasty

C.

MCL reconstruction

D.

femoral chondroplasty

A.

ACL reconstruction

Unused culture tubes are placed on the back table prior to incision. Which of the following should the surgical first assistant do NEXT?

A.

Swab the affected area.

B.

Obtain new culture swabs.

C.

Pass the culture swabs to the surgeon.

D.

Remove tops of tubes in preparation for culture.

B.

Obtain new culture swabs.

During a low anterior resection, the iliac artery is transected. Which of the following is the MOST appropriate instrument to achieve hemostasis?

A.

Debakey angled vascular clamp

B.

Heaney clamp

C.

Rochester Pean clamp

D.

Bulldog clamp

A.

Debakey angled vascular clamp

When the surgical first assistant is placing the second port during a diagnostic laparoscopy, which of the following is the MOST important step?

A.

skin incision

B.

direct visualization

C.

trocar selection

D.

patient positioning

B.

direct visualization

A patient presents with a thigh abscess that was cultured and diagnosed as necrotizing fasciitis. Which of the following is the appropriate treatment?

A.

Pack the wound.

B.

Place a drain and close the wound.

C.

Close the wound.

D.

Debride all the necrotic tissue.

D.

Debride all the necrotic tissue.

Which of the following muscles is used to re-implant a section of the parathyroid if it is inadvertently removed during a thyroidectomy?

A.

sternohyoid

B.

platysma

C.

sternocleidomastoid

D.

sternothyroid

C.

sternocleidomastoid

Before the initiation of cardiopulmonary bypass, it is important that the ACT (Activated Clotting Time) is greater than

A.

100 seconds.

B.

400 seconds.

C.

500 seconds.

D.

1000 seconds.

C.

500 seconds.

During a thoracotomy for tumor resection, the surgeon wants to irrigate with a solution that will cause tumor cells to lyse. Which of the following should the surgical first assistant use?

A.

sterile water

B.

normal saline

C.

glycine

D.

sorbitol

A.

sterile water

A patient presents with extreme right lower quadrant pain, vomiting, and fever. A CT scan indicates free fluid in the abdomen and the extreme pain subsides. Which of the following is the MOST likely surgical intervention?

A.

open appendectomy

B.

laparoscopic appendectomy

C.

exploratory celiotomy

D.

laparoscopic cholecystectomy

A.

open appendectomy

In minimally invasive surgery, which of the following would involve Cartesian coordinate geometry?

A.

robotic endowrist

B.

total joint design

C.

synthetic heart valve manufacture

D.

alignment of compound facial fractures

A.

robotic endowrist

Which of the following is a direct complication resulting from a total laryngectomy with a radical neck dissection?

A.

pneumothorax

B.

cranial aneurysm

C.

ischemic stroke

D.

salivary fistula

D.

salivary fistula

Following an arteriovenous fistula procedure, a patient develops a condition known as “steal syndrome.” Which of the following is the MOST appropriate action to remedy this condition?

A.

Revise fistula.

B.

Continue anticoagulant therapy

C.

Discontinue anticoagulant therapy.

D.

Revise fistula using an argyle shunt.

A.

Revise fistula.

Which of the following is the reason a surgeon would choose to use a locking plate rather than a non-locking plate when repairing a bone fracture?

A.

low bone density

B.

comminuted fracture

C.

compound fracture

D.

high bone density

A.

low bone density

After a coronary artery bypass procedure, the patient is having difficulty transitioning off the cardiopulmonary bypass machine. Which of the following is the MOST frequently used intervention to address this situation?

A.

ventricular assist device

B.

high dose epinephrine

C.

intra-aortic balloon pump (IABP)

D.

extracorporeal membrane oxygenation (ECMO)

C.

intra-aortic balloon pump (IABP)

A patient requires breast reconstructive surgery for breast cancer. When the surgeon places a tissue expander, which of the following muscles are lifted to allow for the placement of the expander?

A.

pectoralis major, deltoid, and superior rectus sheath

B.

pectoralis minor, serratus anterior, and superior rectus sheath

C.

pectoralis major, serratus anterior, and superior rectus sheath

D.

pectoralis minor, serratus anterior, and latissimus dorsi

C.

pectoralis major, serratus anterior, and superior rectus sheath

Which of the following conditions may be induced when thrombin is used in the management of hemostasis during a vascular procedure?

A.

thalassemia

B.

disseminated intravascular clotting

C.

thrombocytopenia

D.

von Willebrand’s disease

B.

disseminated intravascular clotting

A patient presents with a traumatic soft tissue injury with a Class III wound. Which of the following is the MOST appropriate dressing?

A.

Dermabond

B.

Telfa

C.

Tegaderm

D.

wound VAC

D.

wound VAC

Which of the following is an example of a secondary dressing?

A.

ACE bandage

B.

4 X 4 sponge

C.

wound VAC

D.

Telfa

A.

ACE bandage

Which type of drain is generally used for large open contaminated wounds such as perirectal, perianal fistulas, and subcutaneous abscess cavities?

A.

Penrose

B.

Jackson-Pratt

C.

Hemovac

D.

Blake

A.

Penrose

Which of the following is a CONTRAINDICATION for the use of negative-pressure wound therapy?

A.

skin grafts

B.

pressure ulcers

C.

skin flaps

D.

untreated osteomyelitis

D.

untreated osteomyelitis

Which of the following splints would be placed on a patient immediately postoperative for an ORIF Olecranon fracture?

A.

long arm

B.

short arm

C.

long leg

D.

short leg

A.

long arm

If operating with multiple sharps on the surgical field, which of the following is the MOST effective way to reduce potential injuries?

A.

Pass all sharps by hand.

B.

Keep all sharps on the mayo stand.

C.

Place the count board on the mayo stand.

D.

Utilize a neutral zone.

D.

Utilize a neutral zone.

A patient presented in the OR with a cervical spine fracture. Which of the following traction devices is MOST appropriate?

A.

Gardner-Wells tongs

B.

Thomas splint

C.

traction splint

D.

Buck’s traction

A.

Gardner-Wells tongs

Which of the following is the PRIMARY reason an elastic bandage would be utilized on an extremity undergoing a skin graft?

A.

prevention of fluid accumulation at graft site

B.

post-op swelling of the surrounding tissues

C.

securement of underlying bulky dressings

D.

absorption of excess exudate and drainage

A.

prevention of fluid accumulation at graft site

Language barriers with a patient can be a stress-producing situation. One way to help ease their anxiety is to

A.

speak slowly and loudly.

B.

use hand gestures.

C.

speak to their family members.

D.

use a gentle touch.

D.

use a gentle touch.

Who makes medical decisions for a patient with a living will?

A.

the surgeon

B.

the patient

C.

the patient’s nearest living relative

D.

the social worker

B.

the patient

Which of the following forms of radiation is dispersed from a C-arm?

A.

complex

B.

non-ionizing

C.

columnar

D.

ionizing

D.

ionizing

During the timeout, why are all OR personnel required to stop all activity?

A.

to effectively communicate patient identification issues or concerns

B.

to allow the prep solution to dry

C.

to allow equipment to be set up for procedure

D.

to allow anesthesia to anesthetize the patient

A.

to effectively communicate patient identification issues or concerns

For efficiency and safety, the MOST appropriate approach for the surgical first assistant to indicate that the long instrument may be needed and should be opened while preparing for an exploratory laparotomy on an obese patient is to

A.

determine that it is needed and open the long instrument set that the CST had on hold.

B.

consult with the CST in order to have the long instrument set placed on the field now.

C.

wait until the circulator returns from pre-op and ask him/her to have the set counted and on the field.

D.

have the surgeon determine the need for the long instrument set after the procedure begins.

B.

consult with the CST in order to have the long instrument set placed on the field now.

A patient in the preoperative holding area has questions about a surgical procedure. The surgical first assistant should

A.

ask the surgeon to speak with the patient.

B.

answer the questions.

C.

proceed with the procedure.

D.

ask the anesthesia provider to answer the questions.

A.

ask the surgeon to speak with the patient.

The surgical team is beginning their timeout. The x-ray tech walks in and states he is going to leave C-Arm outside the OR. Which of the following should the surgical team do NEXT?

A.

Continue with the timeout.

B.

Tell the x-ray tech OK and continue with timeout.

C.

Start over with the timeout.

D.

Tell the x-ray tech to be quiet or get out.

C.

Start over with the timeout.

To help reduce the risk of an electrostatic spark, which of the following should be done?

A.

Maintain relative humidity in the correct range.

B.

Ask the circulator to move the electrocautery machine farther away from the field.

C.

Wait until the patient is asleep to plug in the electrocautery and turn it on.

D.

Use synthetic fiber blankets.

A.

Maintain relative humidity in the correct range.

In the event of a chemical spill in the OR, which of the following is the MOST appropriate reference for exposure control and clean up?

A.

OSHA manual

B.

chemical label

C.

JCAHO standards

D.

Safety Data Sheets

D.

Safety Data Sheets

If the OR’s humidity increases above 70% before the patient arrives to the OR, which of the following steps should be taken?

A.

Tear down the sterile field and reset once the humidity level is normal.

B.

Proceed as normal.

C.

Monitor room and proceed once the humidity has reached normal levels.

D.

Rinse all instrumentation in saline.

A.

Tear down the sterile field and reset once the humidity level is normal.

During a radical mastectomy, in addition to providing exposure, it is imperative that the surgical first assistant

A.

inquires if a local anesthetic is available for the end of the procedure.

B.

ensures saline is on the field for irrigation.

C.

weighs the specimen.

D.

evacuates the smoke plume.

D.

evacuates the smoke plume.

A skin tag is removed from a patient under general anesthesia. It is not part of the procedure and is not within the surgical site. Under which of the following categories does this fall?

A.

negligence

B.

malpractice

C.

assault and battery

D.

Borrowed Servant Rule

C.

assault and battery

A patient requires an I&D of the thigh. The team has anticipated using 6 bottles of warm saline and brings them to the OR. During the 2-hour case, only four of the bottles are used while the other two have become room temperature. Which of the following actions should be taken with cool saline?

A.

Return to warmer within one hour.

B.

Immediately return to warmer.

C.

Put with cold saline stock.

D.

Cold saline cannot be used.

D.

Cold saline cannot be used.

Which of the following is an example of a high-level disinfecting agent?

A.

glutaraldehyde

B.

isopropyl alcohol

C.

quaternary ammonium

D.

iodophor solution

A.

glutaraldehyde

Which of the following is the vertical dimension within which objects are seen in focus?

A.

depth of field

B.

stereopsis

C.

working distance

D.

focal length

A.

depth of field

To model bone tissue in plastic and reconstructive surgery, which of the following should be used?

A.

bone wax

B.

rotating burr

C.

oscillating saw

D.

bone cement

B.

rotating burr

An example of “cold” sterilization utilizes which of the following chemicals?

A.

glutaraldehyde

B.

phenol

C.

ethylene oxide

D.

hydrogen peroxide

C.

ethylene oxide

Geobacillis stearothermophilus is the biological indicator used in which of the following sterilizers?

A.

steam

B.

dry heat

C.

ethylene oxide

D.

gas plasma

A.

steam

A tourniquet that displays an error message and fails to inflate should be

A.

turned off and restarted.

B.

used for the case.

C.

removed from service.

D.

reported to the charge nurse.

C.

removed from service.

When using a Dynamic Air removal system, which of the following is the minimum required sterilization time for a wrapped instrument pan at 275ºF?

A.

3 minutes

B.

4 minutes

C.

15 minutes

D.

30 minutes

A.

3 minutes

Which of the following instruments is a bipolar electrosurgical instrument?

A.

Kleppinger forceps

B.

Harmonic scalpel

C.

L-hook cautery

D.

Maryland grasper

A.

Kleppinger forceps

Which of the following instruments is a traumatic laparoscopic dissecting instrument?

A.

Prestige

B.

scissors

C.

Davis and Geck forceps

D.

tenaculum

B.

scissors

After completion of a femoral popliteal bypass, which of the following should be used to detect blood flow to the foot?

A.

CT scan

B.

x-ray

C.

MRI

D.

Doppler

D.

Doppler

When a sterilized craniotomy set is opened, a hair is discovered in the set. Which of the following is the MOST appropriate action?

A.

Remove the hair from the set.

B.

Proceed with the case as the set was sterilized.

C.

Replace the set.

D.

Clean affected instruments with betadine.

C.

Replace the set.

Which of the following is the MOST common staple used during a laparoscopic right hemi-colectomy to transect the ilium?

A.

Insorb

B.

EEA

C.

ENDO-GIA

D.

linear

C.

ENDO-GIA

In the instance of dermoid cyst rupture, which of the following irrigants is used to prevent chemical peritonitis caused by the sebaceous material?

A.

water

B.

lactated Ringer’s

C.

normal saline

D.

bacitracin

B.

lactated Ringer’s

Which of the following gases is less soluble in blood and could persist long enough to cause a form of a gas embolus?

A.

carbon dioxide

B.

argon

C.

nitrogen

D.

krypton

B.

argon

When placing a patient prone using chest rolls, which direction should the breasts be diverted?

A.

inward

B.

outward

C.

upward

D.

downward

A.

inward

When placing a patient in the lateral position, right side down with the aid of a Vac-Pac bean bag, which of the following structures is compressed, impairing blood flow?

A.

common iliac

B.

vena cava

C.

great saphenous vein

D.

aorta

B.

vena cava

Using a mesher on a split-thickness skin graft allows for

A.

greater coverage of the wound.

B.

better-looking results postoperatively.

C.

a smooth surface graft.

D.

retained serum on the recipient site.

A.

greater coverage of the wound.

Winged scapula is a condition brought on by an injury to which of the following nerves?

A.

axillary

B.

medial pectoral

C.

long thoracic

D.

thoracodorsal

C.

long thoracic

An autoimmune disease that causes diffuse, bilateral enlargement of the thyroid gland, and hyperthyroidism is

A.

Hashimoto’s thyroiditis.

B.

goiter disease.

C.

Graves’ disease.

D.

malignant disease.

C.

Graves’ disease.

During an open reduction and internal fixation of a clavicular fracture, the surgeon must use extreme caution when drilling screw holes to avoid injury to which of the following structures, which lies directly underneath the clavicle?

A.

subclavian vein

B.

subclavian artery

C.

brachial plexus

D.

aortic arch

A.

subclavian vein

Compression of which of the following structures is considered a surgical emergency with a hemorrhage after a thyroidectomy?

A.

jugular vein

B.

carotid artery

C.

trachea

D.

esophagus

C.

trachea

The gallbladder drains into which of the following structures?

A.

thoracic duct

B.

hepatic duct

C.

common bile duct

D.

cystic duct

D.

cystic duct

During a laparoscopic cholecystectomy, the infundibulum of the gallbladder is retracted to expose the

A.

artery of Drummond.

B.

ligament of Treitz.

C.

triangle of Calot.

D.

sphincter of Oddi.

C.

triangle of Calot.

A distinguishing feature of lupus is

A.

fatigue.

B.

joint pain.

C.

muscle pain.

D.

a butterfly rash.

D.

a butterfly rash.

Which of the following hemodynamic concepts describes the resistance the heart must overcome to pump blood into the systemic circulation?

A.

cardiac output

B.

preload

C.

contractility

D.

afterload

D.

afterload

The third ventricle empties into the fourth ventricle through which of the following small ducts, which is the only exit for the third and lateral ventricles?

A.

foramen of Monro

B.

aqueduct of Sylvius

C.

foramina of Luschka

D.

foramina of Magendie

B.

aqueduct of Sylvius

Which of the following types of hernia consists of a protrusion of fat through defects in the abdominal wall located between the umbilicus and the xiphoid process and symptoms of nausea and slight upper abdominal pain?

A.

spigelian

B.

epigastric

C.

interparietal

D.

umbilical

B.

epigastric

Which of the following bones is the MOST superior tarsal and articulates with the distal end of the tibia?

A.

calcaneus

B.

navicular

C.

talus

D.

cuboid

C.

talus

A latissimus dorsi flap used for breast reconstruction following a mastectomy gets its main blood supply from which of the following arteries?

A.

thoracodorsal

B.

long thoracic

C.

superior epigastric

D.

inferior epigastric

A.

thoracodorsal

A surgeon is performing a lumbar laminectomy and discectomy for an L5-S1 disc herniation that has protruded into the epidural space. Which of the following parts of the disc should be removed?

A.

nucleus pulposus

B.

annulus fibrosus

C.

anterior annulus

D.

anterior ligament

A.

nucleus pulposus

During heart transplant surgery, which of the following is the correct order of chamber/vessel anastomosis?

A.

right atrium, left atrium, aorta, pulmonary artery

B.

left atrium, right atrium, aorta, pulmonary artery

C.

aorta, pulmonary artery, left atrium, right atrium

D.

left atrium, right atrium, pulmonary artery, aorta

D.

left atrium, right atrium, pulmonary artery, aorta

During a Nissen fundoplication, care should be taken when placing medial traction on the stomach to prevent avulsion of the

A.

splenic artery.

B.

gastric arteries.

C.

common duct.

D.

pancreatic duct.

B.

gastric arteries.

To reconstruct the pelvis after prolapse, which of the following anatomical structures could be used as a supportive repair in conjunction with the repair of a rectocele?

A.

transversalis fascia

B.

pelvic diaphragm

C.

levator ani

D.

perivaginal fascia

C.

levator ani

During chest tube placement, it is MOST important to place the tube in the inferior aspect of the intercostal space to avoid

A.

neurovascular injury.

B.

pneumothorax.

C.

lung injury.

D.

rib fracture.

A.

neurovascular injury.

When performing a culdoscopy, the Sims tenaculum should be placed on which of the following aspects of the cervix to assist with visualization of the operative area?

A.

medial

B.

posterior

C.

anterior

D.

lateral

B.

posterior

During a carotid endarterectomy, the patient’s heart rate drops to 30 beats/min. This is likely due to stimulation of the

A.

hypoglossal nerve.

B.

vagus nerve.

C.

jugular vein.

D.

carotid artery.

B.

vagus nerve.

Which of the following prostate procedures involves a transvesical approach?

A.

simple retropubic

B.

suprapubic

C.

radical perineal

D.

laparoscopic

B.

suprapubic

A 47-year-old female is recovering nicely from a laparoscopic Roux-en-Y gastric bypass procedure. On her first day at home following discharge from the hospital, she develops sudden-onset anterior chest pain and shortness of breath and begins sweating profusely. Upon returning to the hospital emergency room, her ECG shows tachycardia and her chest x-ray appears normal. Which of the following is the MOST likely reason for her symptoms?

A.

acute respiratory distress syndrome (ARDS)

B.

acute myocardial infarction

C.

pulmonary embolism

D.

nosocomial pneumonia

C.

pulmonary embolism

A 35-year-old male presents with headache, palpitations, and diaphoresis brought on by micturition. The patient’s identical twin brother died of an undiagnosed sudden death. Which of the following diagnoses is MOST likely?

A.

pheochromocytoma

B.

renal malignancy

C.

pituitary tumor

D.

lymphoma

A.

pheochromocytoma

A 68-year-old female presents with hip pain. She was treated 15 years ago with radiation and chemotherapy for lobular breast cancer. Which of the following is the MOST likely diagnosis?

A.

pathologic hip fracture

B.

traumatic hip fracture

C.

trochanteric bursitis

D.

osteoarthritis

A.

pathologic hip fracture

Which of the following is the goal of a supraglottic laryngectomy to preserve?

A.

ability to use the esopheal voice

B.

ability to speak with a tracheostomy

C.

natural ability to speak

D.

ability to taste food

C.

natural ability to speak

A patient presents in the ER with a bowel obstruction due to a recto-anal mass. Which of the following surgical procedures can be used for a complete resection of the mass?

A.

low anterior resection

B.

transverse colectomy

C.

abdominal perineal resection

D.

right hemi-colectomy

C.

abdominal perineal resection

A patient arrives in the ER following a ground level fall with changes in personality and movement. The patient’s CT scan shows a subdural hematoma. In which of the following lobes is the MOST likely location of the hematoma?

A.

parietal

B.

frontal

C.

occipital

D.

temporal

B.

frontal

A patient presents with a large benign tumor located 3 cm proximal to the anal verge. The surgical first assistant should assist the team in preparing for a

A.

low anterior resection.

B.

anoperineal repair.

C.

total colectomy.

D.

hemorrhoidectomy.

A.

low anterior resection.

A patient undergoing an aortic valve replacement is having difficulty converting to NSR (normal sinus rhythm) and cannot be weaned from cardiopulmonary bypass. Continued failure of the heart to beat correctly after aortic valve replacement is MOST commonly caused by

A.

occlusion of the right coronary artery.

B.

high blood level potassium.

C.

remaining cardioplegia.

D.

defective defibrillator paddles.

A.

occlusion of the right coronary artery.

When performing a hysterectomy and ligating the broad ligament, which of the following vascular structures lies within the broad ligament?

A.

internal iliac artery

B.

external iliac artery

C.

uterine artery

D.

medial superior genicular artery

C.

uterine artery

During laparoscopic pelviscopy, a trocar is accidentally placed into the urinary bladder. The surgical first assistant should anticipate repair of which smooth muscle layer?

A.

adventitia

B.

vesical plexus

C.

detrusor

D.

epithelium

C.

detrusor

A surgeon is performing an arthroscopic ACL reconstruction and needs precise placement of the graft. Which of the following would aid in correct graft placement?

A.

drill sleeve

B.

Hohmann retractors

C.

tension isometer

D.

interference screws

C.

tension isometer

A 91-year-old woman presents to the OR with a hip fracture. Efforts should be made to correct the fracture as soon as possible to avoid complications related to

A.

sartorius muscle injury.

B.

thrombophlebitis.

C.

lateral inferior genicular artery avulsion.

D.

peroneal nerve injury.

B.

thrombophlebitis.

While trying to arrest the heart during an aortic valve replacement surgery, the surgical first assistant notices that the heart is not responding to the instillation of cardioplegia and the left ventricle is distending. Which of the following is the MOST likely cause of this situation?

A.

incorrect cardioplegia solution

B.

aortic insufficiency

C.

aortic dissection

D.

coronary artery disease

B.

aortic insufficiency

Which skin cancer is MOST likely to require the use of a radioactive isotope tracing in conjunction with isosulfan blue sentinel node mapping and excision?

A.

fibrosarcoma

B.

basal cell carcinoma

C.

squamous cell carcinoma

D.

malignant melanoma

D.

malignant melanoma

Which of the following is a potentially lethal pathogen that causes surgical site infections and spreads via the lymphatic system?

A.

Streptococcus pyogenes

B.

Pseudomonas aeruginosa

C.

Clostridium perfringens

D.

Salmonella enterica

A.

Streptococcus pyogenes

Which of the following bacteria is found in the lining of the colon?

A.

Staphylococcus aureus

B.

Escherichia coli

C.

Streptococcus pyogenes

D.

Serratia marcescens

B.

Escherichia coli

Which of the following includes Clostridium difficile, Clostridium perfringens, and Bacteroides fragilis?

A.

gram-negative

B.

aerobes

C.

gram-positive

D.

anaerobes

D.

anaerobes

Which of the following is the MOST prevalent microorganism that causes osteomyelitis?

A.

Escherichia coli

B.

Staphylococcus aureus

C.

Clostridium difficile

D.

Candida albicans

B.

Staphylococcus aureus

Which of the following is the MOST common organism found in necrotizing fasciitis?

A.

Streptococci other than group A

B.

Beta hemolytic streptococcus

C.

Vibrio

D.

Escherichia coli

B.

Beta hemolytic streptococcus

Which of the following is the surgical wound classification for a laparoscopic pyeloplasty?

A.

Class I

B.

Class II

C.

Class III

D.

Class IV

B.

Class II

Which of the following can convert an MRSA-negative patient to an MRSA-positive patient in the advanced age population?

A.

Vancomycin

B.

Cefazolin

C.

Kanamycin

D.

Bacitracin

A.

Vancomycin

During an emergency splenectomy, the surgical first assistant sustains a needle-stick injury. Which of the following are the MOST appropriate actions to take?

A.

Change gloves and wash wound with betadine.

B.

Break scrub and wash wound.

C.

Remain in sterile field and change gloves.

D.

Inquire about patient’s social and medical history.

B.

Break scrub and wash wound.

Preoperative antibiotic prophylaxis should be administered at which of the following points of a surgical procedure?

A.

at the time of the first incision

B.

30 minutes before incision

C.

15 minutes before the incision is closed

D.

at the time of the timeout

B.

30 minutes before incision

Which local anesthetic has the longest analgesic effect?

A.

bupivacaine

B.

lidocaine

C.

procaine

D.

mepivacaine

A.

bupivacaine

Prior to liposuction, 40 mL 1% lidocaine with epinephrine 1:100,000 and 1,000 mL saline are combined to create which solution?

A.

tumescent

B.

papaverine

C.

Dakin’s

D.

Lugol’s

A.

tumescent

During induction of anesthesia, the administration of propofol (Diprivan®) would cause

A.

a hypnotic state and sedation to unconsciousness.

B.

thinning of the blood and prevention of clotting.

C.

relaxation of muscles and dilation of arteries.

D.

relaxation of the bladder and production of urine.

A.

a hypnotic state and sedation to unconsciousness.

Comparatively, which of the following patients is at greatest risk for anesthesia complication during the operative phase?

A.

a patient who is undernourished

B.

a patient who is immunosuppressed

C.

a patient with COPD

D.

a patient with diabetes

C.

a patient with COPD

If a patient does not wish to receive an allogenic blood transfusion, which of the following is an available option to limit the risk of intra-operative blood loss?

A.

Use Dextrose 5% in 0.9% NACL for an IV drip.

B.

Give the patient factor 7 intra-operatively.

C.

Use preoperative autologous blood donation.

D.

Use FFP instead of PRBC’s.

C.

Use preoperative autologous blood donation.

An anesthesiologist is performing spinal anesthesia for a total knee replacement. Shortly after the spinal, the patient develops shortness of breath and respiratory distress. Which of the following has MOST likely happened?

A.

postdural puncture headache

B.

hypotension

C.

total spinal anesthesia

D.

caudal anesthesia

C.

total spinal anesthesia

Which of the following analgesic drugs produces amnesia?

A.

ketamine

B.

propofol

C.

etomidate

D.

midazolam

A.

ketamine

Which of the following is the MAXIMUM dosage of bupivacaine without epinephrine?

A.

150 mg

B.

175 mg

C.

200 mg

D.

225 mg

B.

175 mg

In pre-op, the patient denies taking any anticoagulants. In passing, the patient’s family mentions herbal remedies that the patient has been trying. Which of the following remedies should the surgical first assistant be MOST concerned about causing anticoagulation?

A.

St. John’s wort, cinnamon, and vitamin E

B.

ginseng, licorice, and fish oil

C.

ginseng, kava-kava, and ginger

D.

St. John’s wort, licorice, and fish oil

ginseng, licorice, and fish oil

The topical hemostatic agent thrombin should never be directly injected or allowed to enter into a vessel because

A.

it causes uncontrollable vasospasms.

B.

the effects are reversed when directly injected and cause more severe bleeding.

C.

it can cause clotting and may ultimately lead to death.

D.

the risk of infection increases with direct injection.

C.

it can cause clotting and may ultimately lead to death.

A 16-year-old male comes in for an emergency appendectomy. Surgeon states that the abdominal wall is very rigid and asks if muscle relaxation has been given. The patient is tachycardic and the end-tidal CO2 is rising. What is the MOST likely cause?

A.

sepsis

B.

malignant hyperthermia

C.

acute peritonitis

D.

hypervolemia

B.

malignant hyperthermia

A patient weighing 150 kg is having a malignant hyperthermia crisis. Using 20 mg bottles of Dantrolene, which of the following is the MINIMUM number of bottles that need to be given to the patient?

A.

18 bottles

B.

19 bottles

C.

20 bottles

D.

21 bottles

B.

19 bottles

Which of the following is the MOST effective mechanical method of controlling bleeding occurring from needle holes in vessel anastomoses?
pledget

In which of the following positions is a patient placed for a right nephrectomy?
left lateral kidney

Which of the following verifies that autoclave sterilization has occurred?
a. chemical indicator (incorrect)
b. Julian date
c. heat-sensitive tape
d. biological indicator

During a procedure, a major piece of equipment malfunctions and causes a patient’s death. Which of the following is this situation MOST likely to be classified as?
sentinel event

For which of the following procedures is Gelfoam MOST commonly used?
a. TVH (incorrect)
b. cataract extraction
c. TURP
d. myringotomy

Which of the following test monitors indicate if there is any residual air in the sterilization chamber?
Bowie-Dick test

When exposed to ionizing radiation, staff and patients should wear personal protective equipment containing
lead

Prions are unique from other infectious agents because they lack which of the following?
DNA

In the break room, a precepting surgical technologist is overheard discussing a student’s lack of instrument knowledge. The precepting surgical technologist just committed
slander

During surgery, the surgeon intermittently uses a local anesthetic intraoperatively. The surgical technologist should
keep track of the amounts

Which of the following wounds is MOST likely in the inflammatory phase of wound healing?
an abdominal incision, 20 mins post-closure

The FIRST sign of malignant hyperthermia is
unexplained tachycardia

The surgical incision that allows for the BEST visualization for a Billroth I procedure is
a. McBurney’s
b. Pfannenstiel
c. midline
d. subcostal (incorrect)

When preparing endoscopic cameras for ethylene oxide sterilization, which of the following monitoring processes is utilized to ensure all parameters of sterilization are met?
a. external tape
b. biological indicator
c. dynamic air removal
d. chemical strip (incorrect)

An effective packaging material used for steam sterilization of a Balfour retractor
permits penetration by the sterilant

A surgeon received a report from PACU that a patient is bleeding at the surgical site after a lumbar laminectomy procedure. Upon inspection, the surgeon deems it necessary to return the patient to the OR to correct the issue. Which of the following should the surgeon suspect to occur if the issue is not resolved immediately?
hematoma

Which of the following should be included on an orthopedic set-up?
Key elevator

During dissection of the cystic duct and artery in an open cholecystectomy, which of the following instruments is necessary?
Mixter clamp

Which of the following is a tonsillolith?
a stone within the tonsil

When performing a parotidectomy, which of the following nerves is identified and preserved with the use of a nerve stimulator?
facial

A surgeon has just completed an incisional biopsy and would like to have an immediate diagnosis. The MOST appropriate NEXT step is to send the specimen to pathology in a
a. dry specimen container so that a frozen section can be completed
b. container containing normal saline so that a frozen section can be completed
c. dry specimen container so that a permanent section can be completed
d. specimen container containing normal saline so that a permanent section can be completed (incorrect)

When preparing surgical specimens, in addition to the surgical technologist, who else is responsible for accurately labeling the specimen?
surgeon and circulating nurse

The anterior chamber of the eye is located between the cornea and the
a. posterior chamber
b. lens (incorrect)
c. pupil
d. iris

If a surgeon requests Gelfoam soaked with thrombin, the method to maintain hemostasis is
chemical

A specimen obtained for frozen section is generally removed from the sterile field intraoperatively because it
a. needs to be labeled by the circulator
b. is sent to pathology immediately
c. will contaminate the sterile field
d. needs to be placed in formalin solution (incorrect)

Which of the following is a preferred retractor for a groin incision?
Weitlaner

When infection is present, the MOST commonly used suture is
monofilament

If unexpected heavy blood loss occurs during an abdominal procedure, a surgical technologist should request additional
lap sponges

A patient having recently undergone a laparoscopic cholecystectomy has been transferred to the PACU. The patient becomes tachycardic and the nurse reports a rise in core body temperature. The nurse attempts to communicate with the patient and notices that the patient’s muscles are very rigid. The nurse notifies the surgical team. The surgical technologist should recognize these as signs of
malignant hyperthermia

Herniation of the rectum through a weakened posterior vaginal wall is known as
a rectocele

When steam comes into an autoclave from an outside source, the steam is heated in the
a. vacuum system
b. chamber
c. thermostatic trap (incorrect)
d. jacket

Four people are needed to transfer a patient to the stretcher. Who gives direction for this move?
anesthesiologist

A three-way Foley catheter is inserted following a transurethral resection of the prostate (TURP) in order to
irrigate and aid in hemostasis

The opening in the occipital bone which the spinal cord passes through is the foramen
magnum

The MINIMUM number of surgical team members required to transfer an anesthetized patient from the OR table to the gurney is
four

Which of the following conditions should be treated FIRST?
obstructed airway

When a cholecystectomy is performed, which of the following anatomical structures must be preserved?
cystic artery

Which of the following is used to cross-clamp the intestine during a bowel resection?
a. Allen
b. Lahey (incorrect)
c. Heaney
d. Duval

During a laser laryngoscopy, which of the following supplies must be available?
wet sponges and towels

A patient is scheduled for laser removal of condyloma acuminata. As a safety precaution, the surgical technologist should have which of the following special equipment open and available?
smoke evacuator

Which of the following is considered a contaminated area when evaluating a patient’s skin for the preoperative prep?
a. groin (incorrect)
b. toes
c. body hair
d. draining sinus

A surgical incision that is sutured together so that the wound edges are in direct contact with one another is referred to as which of the following types of closure?
primary intention

A plastic surgeon performs surgery PRIMARILY on which of the following anatomical systems?
integumentary

After completion of a procedure, the nurse notices that the arms were extended beyond 90 degrees. This may have caused injury to the
brachial plexus

Apart from trauma cases, which of the following incisions would be BEST for sigmoid surgery?
a. vertical midline
b. vertical paramedian
c. oblique subcostal
d. lower transverse (incorrect)

Which of the following is the MOST common palpable mass in women of reproductive age?
fibrocystic breast change

Which of the following scissors is MOST likely found in a basic hysterectomy set-up?
Jorgenson

Which of the following supplies is part of the standard set-up for a diagnostic knee arthroscopy?
inflow tubing

During a partial nephrectomy, the surgeon asks for a large chromic suture and Gelfoam. The surgeon would like the Gelfoam to be cut into pledget squares and loaded onto the suture. The MAIN purpose for the use of the Gelfoam and suture is to
topically stop bleeding

Preparing instrument sets and packages for sterilization requires a label indicating the
sterilization date, sterilizer ID, load number, and perparer’s initials

Which of the following is MOST commonly used for wound closure in the presence of infection?
a. stainless steel
b. polyglactin 910
c. plain gut
d. silk (incorrect)

A surgeon is preparing to close the abdomen. Which of the following closing supplies should the surgical technologist have ready for closing the fascial layer?
0 suture and toothed tissue forceps

Which of the following should a surgical technologist do to update a surgeon’s preference card?
consult with the surgeon to determine which supplies should be included

Which of the following instruments is used in otorhinolaryngology surgery?
Baron

A patient is in the supine position for a procedure. After the arms are placed on arm boards, padding is placed under the elbows to decrease pressure on which of the following nerves?
ulnar

A large power saw is not functioning. Which of the following is the MOST likely cause?
the battery has not been adequately charged

Which of the following combination of medications are commonly used during a MAC?
nerve conduction blockades with supplemental analgesics, sedatives, or amnesics

An organization that regulates the production of biological sterilization test packs in-house is
a. CDC (incorrect)
b. ANSI
c. AAMI
d. SDS

Which of the following is the name of the biological indicator used for hydrogen peroxide gas plasma?
bacillus atrophaeus

In which of the following situations can a DNR order be continued in an unconscious patient for a surgical procedure?
the patient’s durable POA requests it be continued

The kidneys help regulate the blood levels of several ions, including
sodium, potassium, and calcium

A surgical “time out” must be performed prior to
making the initial incision

Which of the following tags indicates the least critical of patients in a triage situation?
green

During a laparoscopic appendectomy, the appendix is perforated, and contents begin spilling into the abdomen. Which of the following should the surgical technologist anticipate as the NEXT step?
a. suction the fluid from the abdomen
b. continue removing the appendix as planned
c. place a Kelly clamp over the perforation (incorrect)
d. close the wound and terminate the procedure

If there is any question related to handling a specific type of specimen, which of the following departments should be consulted?
pathology

Which of the following situations would present a problem when preparing instruments for sterilization using a paper-plastic peel pack?
the open end of the paper-plastic peel pack has been sealed with tape

Which of the following important considerations should be taken by the surgical technologist when it comes to the suture material for third intention closure?
a. choosing the proper tensile strength of the suture can aid in healing
b. a low-capillary action suture will not retain moisture
c. the handling of suture is more important than the suture material (incorrect)
d. the size of the suture is more important then the suture material

A surgeon asks for warm irrigation after a right hemicolectomy. Which of the following BEST describes the reason for irrigating at this point in the procedure?
to aid visual inspection for leakage

Which of the following items should be disposed of in an appropriate sharps container?
drill pin

How many lobes do the right and left lungs have, respectively?
three, two

After gowning and gloving, a surgeon activates the DuraPrep applicator and performs the initial skin prep for surgery. Which of the following is the BEST action to take at this time?
regown and reglove the surgeon

The rate per hour in air exchanges in a standard OR suite is
a. 5-10 (incorrect)
b. 15-25
c. 30-35
d. 40-50

Which of the following supplies should be pulled by the surgical technologist for a thoracic procedure?
Jackson-Pratt drainage system

Leave a Comment

Scroll to Top